Programs & Examples On #Vms

VMS is any operating system of the VMS family: OpenVMS, VAX/VMS, VAX-11/VMS, and VAX/VMS which run on the VAX, Alpha, and Itanium-based families of computers.

vagrant primary box defined but commands still run against all boxes

The primary flag seems to only work for vagrant ssh for me.

In the past I have used the following method to hack around the issue.

# stage box intended for configuration closely matching production if ARGV[1] == 'stage'     config.vm.define "stage" do |stage|         box_setup stage, \         "10.9.8.31", "deploy/playbook_full_stack.yml", "deploy/hosts/vagrant_stage.yml"     end end 

Can't install laravel installer via composer

Centos 7 with PHP7.2:

sudo yum --enablerepo=remi-php72 install php-pecl-zip

PHP7 : install ext-dom issue

I faced this exact same issue with Laravel 8.x on Ubuntu 20. I run: sudo apt install php7.4-xml and composer update within the project directory. This fixed the issue.

Laravel: PDOException: could not find driver

First check php -m
.If you don't see mysql driver install mysql sudo apt-cache search php-mysql
Your results will be similar to:

php-mysql - MySQL module for PHP [default]


install php- mysql Driver

sudo apt-get install php7.1-mysql

Extension gd is missing from your system - laravel composer Update

I have installed php7, I did the following to solve exactly the same error

sudo apt-get install php7.0-gd
sudo apt-get install php7.0-intl
sudo apt-get install php7.0-xsl

getting error while updating Composer

The good solution for this error please run this command

composer install --ignore-platform-reqs

PHP 7 simpleXML

my experience

  1. get your php version

    php --version

  2. Instal package for your php version

    sudo apt-get install php7.4-xml

  3. Restart apache

    sudo systemctl reload apache2

How to add LocalDB to Visual Studio 2015 Community's SQL Server Object Explorer?

If you are not sure if local db is installed, or not sure which database name you should use to connect to it - try running 'sqllocaldb info' command - it will show you existing localdb databases.

Now, as far as I know, local db should be installed together with Visual Studio 2015. But probably it is not required feature, and if something goes wrong or it cannot be installed for some reason - Visual Studio installation continues still (note that is just my guess). So to be on the safe side don't rely on it will always be installed together with VS.

Bridged networking not working in Virtualbox under Windows 10

Two line answer: For wired connections it will work smoothly, for wireless turn on 'Promiscious mode' if your wireless adapter does not support promiscious mode, here is the link to workaround. Also visit offical oracle virtualbox documentation to see more details here on using bridged connection over wifi.

SQL Developer with JDK (64 bit) cannot find JVM

I had the same problem and solved it by copying the MSVCR100.dll file from sqldeveloper\jdk\jre\bin to the sqldeveloper\sqldeveloper\bin folder.

Credit goes to Erik Anderson from SQL Developer failed to start


Note that different versions of SQL Developer need different versions of MSVCR*.dll. Various comments below have offered which versions worked for them.

Python get current time in right timezone

To get the current time in the local timezone as a naive datetime object:

from datetime import datetime
naive_dt = datetime.now()

If it doesn't return the expected time then it means that your computer is misconfigured. You should fix it first (it is unrelated to Python).

To get the current time in UTC as a naive datetime object:

naive_utc_dt = datetime.utcnow()

To get the current time as an aware datetime object in Python 3.3+:

from datetime import datetime, timezone

utc_dt = datetime.now(timezone.utc) # UTC time
dt = utc_dt.astimezone() # local time

To get the current time in the given time zone from the tz database:

import pytz

tz = pytz.timezone('Europe/Berlin')
berlin_now = datetime.now(tz)

It works during DST transitions. It works if the timezone had different UTC offset in the past i.e., it works even if the timezone corresponds to multiple tzinfo objects at different times.

json: cannot unmarshal object into Go value of type

You JSON doesn't match your struct fields: E.g. "district" in JSON and "District" as the field.

Also: Your Item is a slice type but your JSON is a dict value. Do not mix this up. Slices decode from arrays.

How do I configure php to enable pdo and include mysqli on CentOS?

mysqli is provided by php-mysql-5.3.3-40.el6_6.x86_64

You may need to try the following

yum install php-mysql-5.3.3-40.el6_6.x86_64

How can I change Mac OS's default Java VM returned from /usr/libexec/java_home

I had a similar situation, and the following process worked for me:

  1. In the terminal, type

    vi ~/.profile
    
  2. Then add this line in the file, and save

    export JAVA_HOME=/Library/Java/JavaVirtualMachines/jdk<version>.jdk/Contents/Home
    

    where version is the one on your computer, such as 1.7.0_25

  3. Exit the editor, then type the following command make it become effective

    source ~/.profile 
    

Then type java -version to check the result

    java -version 

What is .profile? From:http://computers.tutsplus.com/tutorials/speed-up-your-terminal-workflow-with-command-aliases-and-profile--mac-30515

.profile file is a hidden file. It is an optional file which tells the system which commands to run when the user whose profile file it is logs in. For example, if my username is bruno and there is a .profile file in /Users/bruno/, all of its contents will be executed during the log-in procedure.

How do I change the UUID of a virtual disk?

The following worked for me:

  1. run VBoxManage internalcommands sethduuid "VDI/VMDK file" twice (the first time is just to conveniently generate an UUID, you could use any other UUID generation method instead)

  2. open the .vbox file in a text editor

  3. replace the UUID found in Machine uuid="{...}" with the UUID you got when you ran sethduuid the first time

  4. replace the UUID found in HardDisk uuid="{...}" and in Image uuid="{}" (towards the end) with the UUID you got when you ran sethduuid the second time

What are the -Xms and -Xmx parameters when starting JVM?

-Xms initial heap size for the startup, however, during the working process the heap size can be less than -Xms due to users' inactivity or GC iterations. This is not a minimal required heap size.

-Xmx maximal heap size

write a shell script to ssh to a remote machine and execute commands

There is are multiple ways to execute the commands or script in the multiple remote Linux machines. One simple & easiest way is via pssh (parallel ssh program)

pssh: is a program for executing ssh in parallel on a number of hosts. It provides features such as sending input to all of the processes, passing a password to ssh, saving the output to files, and timing out.

Example & Usage:

Connect to host1 and host2, and print "hello, world" from each:

 pssh -i -H "host1 host2" echo "hello, world"

Run commands via a script on multiple servers:

pssh -h hosts.txt -P -I<./commands.sh

Usage & run a command without checking or saving host keys:

pssh -h hostname_ip.txt -x '-q -o StrictHostKeyChecking=no -o PreferredAuthentications=publickey -o PubkeyAuthentication=yes' -i  'uptime; hostname -f'

If the file hosts.txt has a large number of entries, say 100, then the parallelism option may also be set to 100 to ensure that the commands are run concurrently:

pssh -i -h hosts.txt -p 100 -t 0 sleep 10000

Options:
-I: Read input and sends to each ssh process.
-P: Tells pssh to display output as it arrives.
-h: Reads the host's file.
-H : [user@]host[:port] for single-host.
-i: Display standard output and standard error as each host completes
-x args: Passes extra SSH command-line arguments
-o option: Can be used to give options in the format used in the configuration file.(/etc/ssh/ssh_config) (~/.ssh/config)
-p parallelism: Use the given number as the maximum number of concurrent connections
-q Quiet mode: Causes most warning and diagnostic messages to be suppressed.
-t: Make connections time out after the given number of seconds. 0 means pssh will not timeout any connections

When ssh'ing to the remote machine, how to handle when it prompts for RSA fingerprint authentication.

Disable the StrictHostKeyChecking to handle the RSA authentication prompt.
-o StrictHostKeyChecking=no

Source: man pssh

What are advantages of Artificial Neural Networks over Support Vector Machines?

One thing to note is that the two are actually very related. Linear SVMs are equivalent to single-layer NN's (i.e., perceptrons), and multi-layer NNs can be expressed in terms of SVMs. See here for some details.

Shell Script Syntax Error: Unexpected End of File

I have found that this is sometimes caused by running a MS Dos version of a file. If that's the case dos2ux should fix that.

dos2ux file1 > file2

xls to csv converter

Quoting an answer from Scott Ming, which works with workbook containing multiple sheets:

Here is a python script getsheets.py (mirror), you should install pandas and xlrd before you use it.

Run this:

pip3 install pandas xlrd  # or `pip install pandas xlrd`

How does it works?

$ python3 getsheets.py -h
Usage: getsheets.py [OPTIONS] INPUTFILE

Convert a Excel file with multiple sheets to several file with one sheet.

Examples:

    getsheets filename

    getsheets filename -f csv

Options:
-f, --format [xlsx|csv]  Default xlsx.
-h, --help               Show this message and exit.

Convert to several xlsx:

$ python3 getsheets.py goods_temp.xlsx
Sheet.xlsx Done!
Sheet1.xlsx Done!

All Done!

Convert to several csv:

$ python3 getsheets.py goods_temp.xlsx -f csv
Sheet.csv Done!
Sheet1.csv Done!

All Done!

getsheets.py:

# -*- coding: utf-8 -*-

import click
import os
import pandas as pd


def file_split(file):
    s = file.split('.')
    name = '.'.join(s[:-1])  # get directory name
    return name


def getsheets(inputfile, fileformat):
    name = file_split(inputfile)
    try:
        os.makedirs(name)
    except:
        pass

    df1 = pd.ExcelFile(inputfile)
    for x in df1.sheet_names:
        print(x + '.' + fileformat, 'Done!')
        df2 = pd.read_excel(inputfile, sheetname=x)
        filename = os.path.join(name, x + '.' + fileformat)
        if fileformat == 'csv':
            df2.to_csv(filename, index=False)
        else:
            df2.to_excel(filename, index=False)
    print('\nAll Done!')


CONTEXT_SETTINGS = dict(help_option_names=['-h', '--help'])


@click.command(context_settings=CONTEXT_SETTINGS)
@click.argument('inputfile')
@click.option('-f', '--format', type=click.Choice([
    'xlsx', 'csv']), default='xlsx', help='Default xlsx.')
def cli(inputfile, format):
    '''Convert a Excel file with multiple sheets to several file with one sheet.

    Examples:

    \b
        getsheets filename

    \b
        getsheets filename -f csv
    '''
    if format == 'csv':
        getsheets(inputfile, 'csv')
    else:
        getsheets(inputfile, 'xlsx')


cli()

What is the difference between buffer and cache memory in Linux?

Cited answer (for reference):

Short answer: Cached is the size of the page cache. Buffers is the size of in-memory block I/O buffers. Cached matters; Buffers is largely irrelevant.

Long answer: Cached is the size of the Linux page cache, minus the memory in the swap cache, which is represented by SwapCached (thus the total page cache size is Cached + SwapCached). Linux performs all file I/O through the page cache. Writes are implemented as simply marking as dirty the corresponding pages in the page cache; the flusher threads then periodically write back to disk any dirty pages. Reads are implemented by returning the data from the page cache; if the data is not yet in the cache, it is first populated. On a modern Linux system, Cached can easily be several gigabytes. It will shrink only in response to memory pressure. The system will purge the page cache along with swapping data out to disk to make available more memory as needed.

Buffers are in-memory block I/O buffers. They are relatively short-lived. Prior to Linux kernel version 2.4, Linux had separate page and buffer caches. Since 2.4, the page and buffer cache are unified and Buffers is raw disk blocks not represented in the page cache—i.e., not file data. The Buffers metric is thus of minimal importance. On most systems, Buffers is often only tens of megabytes.

How to SSH to a VirtualBox guest externally through a host?

For Windows host, you can :

  1. In virtualbox manager:
    1. select ctrl+G in your virtualbox manager,
    2. then go to network pannel
    3. add a private network
      1. make sure that activate DHCP is NOT selected
  2. In network management (windows)
    1. Select the newly created virtualbox host only adapter and the physical network card
    2. Right-Click and select "Make bridge"
  3. Enjoy

"VT-x is not available" when I start my Virtual machine

You might try reducing your base memory under settings to around 3175MB and reduce your cores to 1. That should work given that your BIOS is set for virtualization. Use the f12 key, security, virtualization to make sure that it is enabled. If it doesn't say VT-x that is ok, it should say VT-d or the like.

Where does Java's String constant pool live, the heap or the stack?

The answer is technically neither. According to the Java Virtual Machine Specification, the area for storing string literals is in the runtime constant pool. The runtime constant pool memory area is allocated on a per-class or per-interface basis, so it's not tied to any object instances at all. The runtime constant pool is a subset of the method area which "stores per-class structures such as the runtime constant pool, field and method data, and the code for methods and constructors, including the special methods used in class and instance initialization and interface type initialization". The VM spec says that although the method area is logically part of the heap, it doesn't dictate that memory allocated in the method area be subject to garbage collection or other behaviors that would be associated with normal data structures allocated to the heap.

how to output every line in a file python

You could try this. It doesn't read all of f into memory at once (using the file object's iterator) and it closes the file when the code leaves the with block.

if data.find('!masters') != -1:
    with open('masters.txt', 'r') as f:
        for line in f:
            print line
            sck.send('PRIVMSG ' + chan + " " + line + '\r\n')

If you're using an older version of python (pre 2.6) you'll have to have

from __future__ import with_statement

Query EC2 tags from within instance

If you are not in the default availability zone the results from overthink would return empty.

ec2-describe-tags \
   --region \
     $(curl -s http://169.254.169.254/latest/meta-data/placement/availability-zone  | sed -e "s/.$//") \
   --filter \
     resource-id=$(curl --silent http://169.254.169.254/latest/meta-data/instance-id)

If you want to add a filter to get a specific tag (elasticbeanstalk:environment-name in my case) then you can do this.

ec2-describe-tags \
   --region \
     $(curl -s http://169.254.169.254/latest/meta-data/placement/availability-zone  | sed -e "s/.$//") \
   --filter \
     resource-id=$(curl --silent http://169.254.169.254/latest/meta-data/instance-id) \
   --filter \
     key=elasticbeanstalk:environment-name | cut -f5

And to get only the value for the tag that I filtered on, we pipe to cut and get the fifth field.

ec2-describe-tags \
  --region \
    $(curl -s http://169.254.169.254/latest/meta-data/placement/availability-zone  | sed -e "s/.$//") \
  --filter \
    resource-id=$(curl --silent http://169.254.169.254/latest/meta-data/instance-id) \
  --filter \
    key=elasticbeanstalk:environment-name | cut -f5

Click outside menu to close in jquery

I think you need something like this: http://jsfiddle.net/BeenYoung/BXaqW/3/

$(document).ready(function() {
  $("ul.opMenu li").each(function(){
      $(this).click(function(){
            if($(this).hasClass('opened')==false){          
                $('.opMenu').find('.opened').removeClass('opened').find('ul').slideUp();
                $(this).addClass('opened'); 
                $(this).find("ul").slideDown();
            }else{
                $(this).removeClass('opened'); 
                $(this).find("ul").slideUp();               
            }
      });
  });    
});

I hope it useful for you!

Getting Java version at runtime

Here's the implementation in JOSM:

/**
 * Returns the Java version as an int value.
 * @return the Java version as an int value (8, 9, etc.)
 * @since 12130
 */
public static int getJavaVersion() {
    String version = System.getProperty("java.version");
    if (version.startsWith("1.")) {
        version = version.substring(2);
    }
    // Allow these formats:
    // 1.8.0_72-ea
    // 9-ea
    // 9
    // 9.0.1
    int dotPos = version.indexOf('.');
    int dashPos = version.indexOf('-');
    return Integer.parseInt(version.substring(0,
            dotPos > -1 ? dotPos : dashPos > -1 ? dashPos : 1));
}

The easiest way to replace white spaces with (underscores) _ in bash

This is borderline programming, but look into using tr:

$ echo "this is just a test" | tr -s ' ' | tr ' ' '_'

Should do it. The first invocation squeezes the spaces down, the second replaces with underscore. You probably need to add TABs and other whitespace characters, this is for spaces only.

Metadata file '.dll' could not be found

I am running Visual Studio 2013.

It appears that the build dependencies were incorrect. Deleting the *.suo files did fix the problems I had.

Java Refuses to Start - Could not reserve enough space for object heap

Given that none of the other suggestions have worked (including many things I'd have suggested myself), to help troubleshoot further, could you try running:

sysctl -a

On both the SuSE and RedHat machines to see if there are any differences? I'm guessing the default configurations are different between these two distributions that's causing this.

Apache Name Virtual Host with SSL

Apache doesn't support SSL on name-based virtual host, only on IP based Virtual Hosts.

Source: Apache 2.2 SSL FAQ question Why is it not possible to use Name-Based Virtual Hosting to identify different SSL virtual hosts?

Unlike SSL, the TLS specification allows for name-based hosts (SNI as mentioned by someone else), but Apache doesn't yet support this feature. It supposedly will in a future release when compiled against openssl 0.9.8.

Also, mod_gnutls claims to support SNI, but I've never actually tried it.

Setting the default Java character encoding

Recently I bumped into a local company's Notes 6.5 system and found out the webmail would show unidentifiable characters on a non-Zhongwen localed Windows installation. Have dug for several weeks online, figured it out just few minutes ago:

In Java properties, add the following string to Runtime Parameters

-Dfile.encoding=MS950 -Duser.language=zh -Duser.country=TW -Dsun.jnu.encoding=MS950

UTF-8 setting would not work in this case.

How to keep a VMWare VM's clock in sync?

If your host time is correct, you can set the following .vmx configuration file option to enable periodic synchronization:

tools.syncTime = true

By default, this synchronizes the time every minute. To change the periodic rate, set the following option to the desired synch time in seconds:

tools.syncTime.period = 60

For this to work you need to have VMWare tools installed in your guest OS.

See http://www.vmware.com/pdf/vmware_timekeeping.pdf for more information

How do I remove a property from a JavaScript object?

Property Removal in JavaScript

There are many different options presented on this page, not because most of the options are wrong—or because the answers are duplicates—but because the appropriate technique depends on the situation you're in and the goals of the tasks you and/or you team are trying to fulfill. To answer you question unequivocally, one needs to know:

  1. The version of ECMAScript you're targeting
  2. The range of object types you want to remove properties on and the type of property names you need to be able to omit (Strings only? Symbols? Weak references mapped from arbitrary objects? These have all been types of property pointers in JavaScript for years now)
  3. The programming ethos/patterns you and your team use. Do you favor functional approaches and mutation is verboten on your team, or do you employ wild west mutative object-oriented techniques?
  4. Are you looking to achieve this in pure JavaScript or are you willing & able to use a 3rd-party library?

Once those four queries have been answered, there are essentially four categories of "property removal" in JavaScript to chose from in order to meet your goals. They are:

Mutative object property deletion, unsafe

This category is for operating on object literals or object instances when you want to retain/continue to use the original reference and aren't using stateless functional principles in your code. An example piece of syntax in this category:

'use strict'
const iLikeMutatingStuffDontI = { myNameIs: 'KIDDDDD!', [Symbol.for('amICool')]: true }
delete iLikeMutatingStuffDontI[Symbol.for('amICool')] // true
Object.defineProperty({ myNameIs: 'KIDDDDD!', 'amICool', { value: true, configurable: false })
delete iLikeMutatingStuffDontI['amICool'] // throws

This category is the oldest, most straightforward & most widely supported category of property removal. It supports Symbol & array indexes in addition to strings and works in every version of JavaScript except for the very first release. However, it's mutative which violates some programming principles and has performance implications. It also can result in uncaught exceptions when used on non-configurable properties in strict mode.

Rest-based string property omission

This category is for operating on plain object or array instances in newer ECMAScript flavors when a non-mutative approach is desired and you don't need to account for Symbol keys:

const foo = { name: 'KIDDDDD!', [Symbol.for('isCool')]: true }
const { name, ...coolio } = foo // coolio doesn't have "name"
const { isCool, ...coolio2 } = foo // coolio2 has everything from `foo` because `isCool` doesn't account for Symbols :(

Mutative object property deletion, safe

This category is for operating on object literals or object instances when you want to retain/continue to use the original reference while guarding against exceptions being thrown on unconfigurable properties:

'use strict'
const iLikeMutatingStuffDontI = { myNameIs: 'KIDDDDD!', [Symbol.for('amICool')]: true }
Reflect.deleteProperty(iLikeMutatingStuffDontI, Symbol.for('amICool')) // true
Object.defineProperty({ myNameIs: 'KIDDDDD!', 'amICool', { value: true, configurable: false })
Reflect.deleteProperty(iLikeMutatingStuffDontI, 'amICool') // false

In addition, while mutating objects in-place isn't stateless, you can use the functional nature of Reflect.deleteProperty to do partial application and other functional techniques that aren't possible with delete statements.

Syntax-based string property omission

This category is for operating on plain object or array instances in newer ECMAScript flavors when a non-mutative approach is desired and you don't need to account for Symbol keys:

const foo = { name: 'KIDDDDD!', [Symbol.for('isCool')]: true }
const { name, ...coolio } = foo // coolio doesn't have "name"
const { isCool, ...coolio2 } = foo // coolio2 has everything from `foo` because `isCool` doesn't account for Symbols :(

Library-based property omission

This category is generally allows for greater functional flexibility, including accounting for Symbols & omitting more than one property in one statement:

const o = require("lodash.omit")
const foo = { [Symbol.for('a')]: 'abc', b: 'b', c: 'c' }
const bar = o(foo, 'a') // "'a' undefined"
const baz = o(foo, [ Symbol.for('a'), 'b' ]) // Symbol supported, more than one prop at a time, "Symbol.for('a') undefined"

How to list the properties of a JavaScript object?

if you are trying to get the elements only but not the functions then this code can help you

this.getKeys = function() {

    var keys = new Array();
    for(var key in this) {

        if( typeof this[key] !== 'function') {

            keys.push(key);
        }
    }
    return keys;
}

this is part of my implementation of the HashMap and I only want the keys, "this" is the hashmap object that contains the keys

How do I find which process is leaking memory?

If you can't do it deductively, consider the Signal Flare debugging pattern: Increase the amount of memory allocated by one process by a factor of ten. Then run your program.

If the amount of the memory leaked is the same, that process was not the source of the leak; restore the process and make the same modification to the next process.

When you hit the process that is responsible, you'll see the size of your memory leak jump (the "signal flare"). You can narrow it down still further by selectively increasing the allocation size of separate statements within this process.

What are some resources for getting started in operating system development?

The x86 JS simulator and ARM simulator can also be very useful to understand how different pieces hardware works and make tests without exiting your favourite browser.

Environment.GetFolderPath(...CommonApplicationData) is still returning "C:\Documents and Settings\" on Vista

Output on Windows 7 (64-bit)

SpecialFolder.CommonApplicationData: C:\ProgramData 
SpecialFolder.CommonDesktopDirectory: C:\Users\Public\Desktop
SpecialFolder.CommonStartMenu: C:\ProgramData\Microsoft\Windows\Start Menu
SpecialFolder.CommonPrograms: C:\ProgramData\Microsoft\Windows\Start Menu\Programs
SpecialFolder.CommonProgramFiles: C:\Program Files\Common Files
SpecialFolder.CommonProgramFilesX86: C:\Program Files (x86)\Common Files
SpecialFolder.CommonStartup: C:\ProgramData\Microsoft\Windows\Start Menu\Programs\Startup
SpecialFolder.ProgramFiles: C:\Program Files
SpecialFolder.ProgramFilesX86: C:\Program Files (x86)
SpecialFolder.System: C:\Windows\system32
SpecialFolder.SystemX86: C:\Windows\SysWOW64

Output on Windows XP

SpecialFolder.CommonApplicationData: C:\Documents and Settings\All Users\Application Data
SpecialFolder.CommonDesktopDirectory: C:\Documents and Settings\All Users\Desktop
SpecialFolder.CommonPrograms: C:\Documents and Settings\All Users\Start Menu\Programs
SpecialFolder.CommonProgramFiles: C:\Program Files\Common Files
SpecialFolder.CommonProgramFilesX86:
SpecialFolder.CommonStartMenu: C:\Documents and Settings\All Users\Start Menu
SpecialFolder.CommonStartup: C:\Documents and Settings\All Users\Start Menu\Programs\Startup
SpecialFolder.ProgramFiles: C:\Program Files
SpecialFolder.ProgramFilesX86:
SpecialFolder.System: C:\WINDOWS\system32
SpecialFolder.SystemX86: C:\WINDOWS\system32

A Generic error occurred in GDI+ in Bitmap.Save method

Create folder path image/thumbs on your hard disk => Problem solved!

Opening a .ipynb.txt File

These steps work for me:

  • Open the file in Jupyter Notebook.
  • Rename the file: Click File > Rename, change the name so that it ends with '.ipynb' behind, and click OK
  • Close the file.
  • From the Jupyter Notebook's directory tree, click the filename to open it.

How does the data-toggle attribute work? (What's its API?)

The data-* attributes is used to store custom data private to the page or application

So Bootstrap uses these attributes for saving states of objects

W3School data-* description

Do I need <class> elements in persistence.xml?

For those running JPA in Spring, from version 3.1 onwards, you can set packagesToScan property under LocalContainerEntityManagerFactoryBean and get rid of persistence.xml altogether.

Here's the low-down

operator << must take exactly one argument

I ran into this problem with templated classes. Here's a more general solution I had to use:

template class <T>
class myClass
{
    int myField;

    // Helper function accessing my fields
    void toString(std::ostream&) const;

    // Friend means operator<< can use private variables
    // It needs to be declared as a template, but T is taken
    template <class U>
    friend std::ostream& operator<<(std::ostream&, const myClass<U> &);
}

// Operator is a non-member and global, so it's not myClass<U>::operator<<()
// Because of how C++ implements templates the function must be
// fully declared in the header for the linker to resolve it :(
template <class U>
std::ostream& operator<<(std::ostream& os, const myClass<U> & obj)
{
  obj.toString(os);
  return os;
}

Now: * My toString() function can't be inline if it is going to be tucked away in cpp. * You're stuck with some code in the header, I couldn't get rid of it. * The operator will call the toString() method, it's not inlined.

The body of operator<< can be declared in the friend clause or outside the class. Both options are ugly. :(

Maybe I'm misunderstanding or missing something, but just forward-declaring the operator template doesn't link in gcc.

This works too:

template class <T>
class myClass
{
    int myField;

    // Helper function accessing my fields
    void toString(std::ostream&) const;

    // For some reason this requires using T, and not U as above
    friend std::ostream& operator<<(std::ostream&, const myClass<T> &)
    {
        obj.toString(os);
        return os;
    }
}

I think you can also avoid the templating issues forcing declarations in headers, if you use a parent class that is not templated to implement operator<<, and use a virtual toString() method.

Add and remove a class on click using jQuery?

Here is an article with live working demo Class Animation In JQuery

You can try this,

$(function () {
   $("#btnSubmit").click(function () {
   $("#btnClass").removeClass("btnDiv").addClass("btn");
   });
});

you can also use switchClass() method - it allows you to animate the transition of adding and removing classes at the same time.

$(function () {
        $("#btnSubmit").click(function () {
            $("#btnClass").switchClass("btn", "btnReset", 1000, "easeInOutQuad");
        });
    });

Sorting list based on values from another list

Another alternative, combining several of the answers.

zip(*sorted(zip(Y,X)))[1]

In order to work for python3:

list(zip(*sorted(zip(B,A))))[1]

Pass a PHP variable value through an HTML form

EDIT: After your comments, I understand that you want to pass variable through your form.

You can do this using hidden field:

<input type='hidden' name='var' value='<?php echo "$var";?>'/> 

In PHP action File:

<?php 
   if(isset($_POST['var'])) $var=$_POST['var'];
?>

Or using sessions: In your first page:

 $_SESSION['var']=$var;

start_session(); should be placed at the beginning of your php page.

In PHP action File:

if(isset($_SESSION['var'])) $var=$_SESSION['var'];

First Answer:

You can also use $GLOBALS :

if (isset($_POST['save_exit']))
{

   echo $GLOBALS['var']; 

}

Check this documentation for more informations.

Adding a view controller as a subview in another view controller

Thanks to Rob. Adding detailed syntax for your second observation :

let controller:MyView = self.storyboard!.instantiateViewControllerWithIdentifier("MyView") as! MyView
controller.ANYPROPERTY=THEVALUE // If you want to pass value
controller.view.frame = self.view.bounds
self.view.addSubview(controller.view)
self.addChildViewController(controller)
controller.didMoveToParentViewController(self)

And to remove the viewcontroller :

self.willMoveToParentViewController(nil)
self.view.removeFromSuperview()
self.removeFromParentViewController() 

For loop for HTMLCollection elements

There's no reason to use es6 features to escape for looping if you're on IE9 or above.

In ES5, there are two good options. First, you can "borrow" Array's forEach as evan mentions.

But even better...

Use Object.keys(), which does have forEach and filters to "own properties" automatically.

That is, Object.keys is essentially equivalent to doing a for... in with a HasOwnProperty, but is much smoother.

var eventNodes = document.getElementsByClassName("events");
Object.keys(eventNodes).forEach(function (key) {
    console.log(eventNodes[key].id);
});

Counting the occurrences / frequency of array elements

One line ES6 solution. So many answers using object as a map but I can't see anyone using an actual Map

const map = arr.reduce((acc, e) => acc.set(e, (acc.get(e) || 0) + 1), new Map());

Use map.keys() to get unique elements

Use map.values() to get the occurrences

Use map.entries() to get the pairs [element, frequency]

_x000D_
_x000D_
var arr = [5, 5, 5, 2, 2, 2, 2, 2, 9, 4]_x000D_
_x000D_
const map = arr.reduce((acc, e) => acc.set(e, (acc.get(e) || 0) + 1), new Map());_x000D_
_x000D_
console.info([...map.keys()])_x000D_
console.info([...map.values()])_x000D_
console.info([...map.entries()])
_x000D_
_x000D_
_x000D_

Showing percentages above bars on Excel column graph

You can do this with a pivot table and add a line with the pourcentage for each category like brettdj showed in his answer. But if you want to keep your data as it is, there is a solution by using some javascript.

Javascript is a powerful language offering a lot of useful data visualization libraries like plotly.js.

Here is a working code I have written for you:

https://www.funfun.io/1/#/edit/5a58c6368dfd67466879ed27

In this example, I use a Json file to get the data from the embedded spreadsheet, so I can use it in my javascript code and create a bar chart.

I calculate the percentage by adding the values of all the category present in the table and using this formula (you can see it in the script.js file): Percentage (%) = 100 x partial value / total value

It automatically calculates the total and pourcentage even if you add more categories.

I used plotly.js to create my chart, it has a good documentation and lots of examples for beginners, this code gets all the option you want to use:

var trace1 = {
    x: xValue, 
    y: data,
    type: 'bar',
    text: yValue,
    textposition: 'auto',
    hoverinfo: 'none',
    marker: {
    color: 'yellow',
    opacity: 0.6,
    line: {
      color: 'yellow',
      width: 1.5
    }
  }
};

It is rather self explanatory, the text is where you put the percentage.

Once you've made your chart you can load it in excel by passing the URL in the Funfun add-in. Here is how it looks like with my example:

final

I know it is an old post but I hope it helps people with the same problem !

Disclosure : I’m a developer of funfun

How to connect from windows command prompt to mysql command line

Following commands will connect to any MySQL database

shell> mysql --host=localhost --user=myname --password=mypass mydb

or

shell> mysql -h localhost -u myname -pmypass mydb

Since it shows the password in plain text, you can type password later as prompted. So, the command will be as follows

shell> mysql --host=localhost --user=myname --password mydb
shell> mysql -h localhost -u myname -p mydb

socket connect() vs bind()

From Wikipedia http://en.wikipedia.org/wiki/Berkeley_sockets#bind.28.29

connect():

The connect() system call connects a socket, identified by its file descriptor, to a remote host specified by that host's address in the argument list.

Certain types of sockets are connectionless, most commonly user datagram protocol sockets. For these sockets, connect takes on a special meaning: the default target for sending and receiving data gets set to the given address, allowing the use of functions such as send() and recv() on connectionless sockets.

connect() returns an integer representing the error code: 0 represents success, while -1 represents an error.

bind():

bind() assigns a socket to an address. When a socket is created using socket(), it is only given a protocol family, but not assigned an address. This association with an address must be performed with the bind() system call before the socket can accept connections to other hosts. bind() takes three arguments:

sockfd, a descriptor representing the socket to perform the bind on. my_addr, a pointer to a sockaddr structure representing the address to bind to. addrlen, a socklen_t field specifying the size of the sockaddr structure. Bind() returns 0 on success and -1 if an error occurs.

Examples: 1.)Using Connect

#include <stdio.h>
#include <sys/socket.h>
#include <netinet/in.h>
#include <string.h>

int main(){
  int clientSocket;
  char buffer[1024];
  struct sockaddr_in serverAddr;
  socklen_t addr_size;

  /*---- Create the socket. The three arguments are: ----*/
  /* 1) Internet domain 2) Stream socket 3) Default protocol (TCP in this case) */
  clientSocket = socket(PF_INET, SOCK_STREAM, 0);

  /*---- Configure settings of the server address struct ----*/
  /* Address family = Internet */
  serverAddr.sin_family = AF_INET;
  /* Set port number, using htons function to use proper byte order */
  serverAddr.sin_port = htons(7891);
  /* Set the IP address to desired host to connect to */
  serverAddr.sin_addr.s_addr = inet_addr("192.168.1.17");
  /* Set all bits of the padding field to 0 */
  memset(serverAddr.sin_zero, '\0', sizeof serverAddr.sin_zero);  

  /*---- Connect the socket to the server using the address struct ----*/
  addr_size = sizeof serverAddr;
  connect(clientSocket, (struct sockaddr *) &serverAddr, addr_size);

  /*---- Read the message from the server into the buffer ----*/
  recv(clientSocket, buffer, 1024, 0);

  /*---- Print the received message ----*/
  printf("Data received: %s",buffer);   

  return 0;
}

2.)Bind Example:

int main()
{
    struct sockaddr_in source, destination = {};  //two sockets declared as previously
    int sock = 0;
    int datalen = 0;
    int pkt = 0;

    uint8_t *send_buffer, *recv_buffer;

    struct sockaddr_storage fromAddr;   // same as the previous entity struct sockaddr_storage serverStorage;
    unsigned int addrlen;  //in the previous example socklen_t addr_size;
    struct timeval tv;
    tv.tv_sec = 3;  /* 3 Seconds Time-out */
    tv.tv_usec = 0;

    /* creating the socket */         
    if ((sock = socket(AF_INET, SOCK_DGRAM, IPPROTO_UDP)) < 0) 
        printf("Failed to create socket\n");

    /*set the socket options*/
    setsockopt(sock, SOL_SOCKET, SO_RCVTIMEO, (char *)&tv, sizeof(struct timeval));

    /*Inititalize source to zero*/
    memset(&source, 0, sizeof(source));       //source is an instance of sockaddr_in. Initialization to zero
    /*Inititalize destinaton to zero*/
    memset(&destination, 0, sizeof(destination));


    /*---- Configure settings of the source address struct, WHERE THE PACKET IS COMING FROM ----*/
    /* Address family = Internet */
    source.sin_family = AF_INET;    
    /* Set IP address to localhost */   
    source.sin_addr.s_addr = INADDR_ANY;  //INADDR_ANY = 0.0.0.0
    /* Set port number, using htons function to use proper byte order */
    source.sin_port = htons(7005); 
    /* Set all bits of the padding field to 0 */
    memset(source.sin_zero, '\0', sizeof source.sin_zero); //optional


    /*bind socket to the source WHERE THE PACKET IS COMING FROM*/
    if (bind(sock, (struct sockaddr *) &source, sizeof(source)) < 0) 
        printf("Failed to bind socket");

    /* setting the destination, i.e our OWN IP ADDRESS AND PORT */
    destination.sin_family = AF_INET;                 
    destination.sin_addr.s_addr = inet_addr("127.0.0.1");  
    destination.sin_port = htons(7005); 

    //Creating a Buffer;
    send_buffer=(uint8_t *) malloc(350);
    recv_buffer=(uint8_t *) malloc(250);

    addrlen=sizeof(fromAddr);

    memset((void *) recv_buffer, 0, 250);
    memset((void *) send_buffer, 0, 350);

    sendto(sock, send_buffer, 20, 0,(struct sockaddr *) &destination, sizeof(destination));

    pkt=recvfrom(sock, recv_buffer, 98,0,(struct sockaddr *)&destination, &addrlen);
    if(pkt > 0)
        printf("%u bytes received\n", pkt);
    }

I hope that clarifies the difference

Please note that the socket type that you declare will depend on what you require, this is extremely important

Count number of cells with any value (string or number) in a column in Google Docs Spreadsheet

The SUBTOTAL function can be used if you want to get the count respecting any filters you use on the page.

=SUBTOTAL(103, A1:A200)

will help you get count of non-empty rows, respecting filters.

103 - is similar to COUNTA, but ignores empty rows and also respects filters.

Reference : SUBTOTAL function

How to convert hex string to Java string?

First of all read in the data, then convert it to byte array:

 byte b = Byte.parseByte(str, 16); 

and then use String constructor:

new String(byte[] bytes) 

or if the charset is not system default then:

new String(byte[] bytes, String charsetName) 

How do emulators work and how are they written?

Emulation is a multi-faceted area. Here are the basic ideas and functional components. I'm going to break it into pieces and then fill in the details via edits. Many of the things I'm going to describe will require knowledge of the inner workings of processors -- assembly knowledge is necessary. If I'm a bit too vague on certain things, please ask questions so I can continue to improve this answer.

Basic idea:

Emulation works by handling the behavior of the processor and the individual components. You build each individual piece of the system and then connect the pieces much like wires do in hardware.

Processor emulation:

There are three ways of handling processor emulation:

  • Interpretation
  • Dynamic recompilation
  • Static recompilation

With all of these paths, you have the same overall goal: execute a piece of code to modify processor state and interact with 'hardware'. Processor state is a conglomeration of the processor registers, interrupt handlers, etc for a given processor target. For the 6502, you'd have a number of 8-bit integers representing registers: A, X, Y, P, and S; you'd also have a 16-bit PC register.

With interpretation, you start at the IP (instruction pointer -- also called PC, program counter) and read the instruction from memory. Your code parses this instruction and uses this information to alter processor state as specified by your processor. The core problem with interpretation is that it's very slow; each time you handle a given instruction, you have to decode it and perform the requisite operation.

With dynamic recompilation, you iterate over the code much like interpretation, but instead of just executing opcodes, you build up a list of operations. Once you reach a branch instruction, you compile this list of operations to machine code for your host platform, then you cache this compiled code and execute it. Then when you hit a given instruction group again, you only have to execute the code from the cache. (BTW, most people don't actually make a list of instructions but compile them to machine code on the fly -- this makes it more difficult to optimize, but that's out of the scope of this answer, unless enough people are interested)

With static recompilation, you do the same as in dynamic recompilation, but you follow branches. You end up building a chunk of code that represents all of the code in the program, which can then be executed with no further interference. This would be a great mechanism if it weren't for the following problems:

  • Code that isn't in the program to begin with (e.g. compressed, encrypted, generated/modified at runtime, etc) won't be recompiled, so it won't run
  • It's been proven that finding all the code in a given binary is equivalent to the Halting problem

These combine to make static recompilation completely infeasible in 99% of cases. For more information, Michael Steil has done some great research into static recompilation -- the best I've seen.

The other side to processor emulation is the way in which you interact with hardware. This really has two sides:

  • Processor timing
  • Interrupt handling

Processor timing:

Certain platforms -- especially older consoles like the NES, SNES, etc -- require your emulator to have strict timing to be completely compatible. With the NES, you have the PPU (pixel processing unit) which requires that the CPU put pixels into its memory at precise moments. If you use interpretation, you can easily count cycles and emulate proper timing; with dynamic/static recompilation, things are a /lot/ more complex.

Interrupt handling:

Interrupts are the primary mechanism that the CPU communicates with hardware. Generally, your hardware components will tell the CPU what interrupts it cares about. This is pretty straightforward -- when your code throws a given interrupt, you look at the interrupt handler table and call the proper callback.

Hardware emulation:

There are two sides to emulating a given hardware device:

  • Emulating the functionality of the device
  • Emulating the actual device interfaces

Take the case of a hard-drive. The functionality is emulated by creating the backing storage, read/write/format routines, etc. This part is generally very straightforward.

The actual interface of the device is a bit more complex. This is generally some combination of memory mapped registers (e.g. parts of memory that the device watches for changes to do signaling) and interrupts. For a hard-drive, you may have a memory mapped area where you place read commands, writes, etc, then read this data back.

I'd go into more detail, but there are a million ways you can go with it. If you have any specific questions here, feel free to ask and I'll add the info.

Resources:

I think I've given a pretty good intro here, but there are a ton of additional areas. I'm more than happy to help with any questions; I've been very vague in most of this simply due to the immense complexity.

Obligatory Wikipedia links:

General emulation resources:

  • Zophar -- This is where I got my start with emulation, first downloading emulators and eventually plundering their immense archives of documentation. This is the absolute best resource you can possibly have.
  • NGEmu -- Not many direct resources, but their forums are unbeatable.
  • RomHacking.net -- The documents section contains resources regarding machine architecture for popular consoles

Emulator projects to reference:

  • IronBabel -- This is an emulation platform for .NET, written in Nemerle and recompiles code to C# on the fly. Disclaimer: This is my project, so pardon the shameless plug.
  • BSnes -- An awesome SNES emulator with the goal of cycle-perfect accuracy.
  • MAME -- The arcade emulator. Great reference.
  • 6502asm.com -- This is a JavaScript 6502 emulator with a cool little forum.
  • dynarec'd 6502asm -- This is a little hack I did over a day or two. I took the existing emulator from 6502asm.com and changed it to dynamically recompile the code to JavaScript for massive speed increases.

Processor recompilation references:

  • The research into static recompilation done by Michael Steil (referenced above) culminated in this paper and you can find source and such here.

Addendum:

It's been well over a year since this answer was submitted and with all the attention it's been getting, I figured it's time to update some things.

Perhaps the most exciting thing in emulation right now is libcpu, started by the aforementioned Michael Steil. It's a library intended to support a large number of CPU cores, which use LLVM for recompilation (static and dynamic!). It's got huge potential, and I think it'll do great things for emulation.

emu-docs has also been brought to my attention, which houses a great repository of system documentation, which is very useful for emulation purposes. I haven't spent much time there, but it looks like they have a lot of great resources.

I'm glad this post has been helpful, and I'm hoping I can get off my arse and finish up my book on the subject by the end of the year/early next year.

CSS 100% height with padding/margin

Frank's example confused me a bit - it didn't work in my case because I didn't understand positioning well enough yet. It's important to note that the parent container element needs to have a non-static position (he mentioned this but I overlooked it, and it wasn't in his example).

Here's an example where the child - given padding and a border - uses absolute positioning to fill the parent 100%. The parent uses relative positioning in order to provide a point of reference for the child's position while remaining in the normal flow - the next element "more-content" is not affected:

#box {
    position: relative;
    height: 300px;
    width: 600px;
}

#box p {
    position: absolute;
    border-style: dashed;
    padding: 1em;
    top: 0;
    right: 0;
    bottom: 0;
    left: 0;
}
<div id="box">
  <p>100% height and width!</p>
</div>
<div id="more-content">
</div>

A useful link for quickly learning CSS positioning

Using an array as needles in strpos

You can iterate through the array and set a "flag" value if strpos returns false.

$flag = false;
foreach ($find_letters as $letter)
{
    if (strpos($string, $letter) === false)
    {
        $flag = true;
    }
}

Then check the value of $flag.

ImportError: No module named model_selection

do you have sklearn? if not, do the following:

sudo pip install sklearn

After installing sklearn

from sklearn.model_selection import train_test_split

works fine

join on multiple columns

The other queries are all going base on any ONE of the conditions qualifying and it will return a record... if you want to make sure the BOTH columns of table A are matched, you'll have to do something like...

select 
      tA.Col1,
      tA.Col2,
      tB.Val
   from
      TableA tA
         join TableB tB
            on  ( tA.Col1 = tB.Col1 OR tA.Col1 = tB.Col2 )
            AND ( tA.Col2 = tB.Col1 OR tA.Col2 = tB.Col2 )

Convert timedelta to total seconds

Use timedelta.total_seconds().

>>> import datetime
>>> datetime.timedelta(seconds=24*60*60).total_seconds()
86400.0

Using Auto Layout in UITableView for dynamic cell layouts & variable row heights

In my case, the padding was because of the sectionHeader and sectionFooter heights, where storyboard allowed me to change it to minimum 1. So in viewDidLoad method:

tableView.sectionHeaderHeight = 0
tableView.sectionFooterHeight = 0

Get first 100 characters from string, respecting full words

This function shortens a string by adding "..." at a word boundary whenever possible. The returned string will have a maximum length of $len including "...".

function truncate($str, $len) {
  $tail = max(0, $len-10);
  $trunk = substr($str, 0, $tail);
  $trunk .= strrev(preg_replace('~^..+?[\s,:]\b|^...~', '...', strrev(substr($str, $tail, $len-$tail))));
  return $trunk;
}

Examples outputs:

  • truncate("Thanks for contributing an answer to Stack Overflow!", 15)
    returns "Thanks for..."
  • truncate("To learn more, see our tips on writing great answers.", 15)
    returns "To learn more..." (comma also truncated)
  • truncate("Pseudopseudohypoparathyroidism", 15)
    returns "Pseudopseudo..."

How to access custom attributes from event object in React?

You can simply use event.target.dataset object . This will give you the object with all data attributes.

How to get the size of a JavaScript object?

This Javascript library sizeof.js does the same thing. Include it like this

<script type="text/javascript" src="sizeof.js"></script>

The sizeof function takes an object as a parameter and returns its approximate size in bytes. For example:

// define an object
var object =
    {
      'boolean' : true,
      'number'  : 1,
      'string'  : 'a',
      'array'   : [1, 2, 3]
    };

// determine the size of the object
var size = sizeof(object);

The sizeof function can handle objects that contain multiple references to other objects and recursive references.

Originally published here.

How to run a bash script from C++ program

Use the system function.

system("myfile.sh"); // myfile.sh should be chmod +x

How to write to file in Ruby?

To destroy the previous contents of the file, then write a new string to the file:

open('myfile.txt', 'w') { |f| f << "some text or data structures..." } 

To append to a file without overwriting its old contents:

open('myfile.txt', "a") { |f| f << 'I am appended string' } 

Make Font Awesome icons in a circle?

You can also do this. I wanted to add a circle around my icomoon icons. Here is the code.

span {
font-size: 54px;
border-radius: 50%;
border: 10px solid rgb(205, 209, 215);
padding: 30px;
}

Call a method of a controller from another controller using 'scope' in AngularJS

The best approach for you to communicate between the two controllers is to use events.

Scope Documentation

In this check out $on, $broadcast and $emit.

In general use case the usage of angular.element(catapp).scope() was designed for use outside the angular controllers, like within jquery events.

Ideally in your usage you would write an event in controller 1 as:

$scope.$on("myEvent", function (event, args) {
   $scope.rest_id = args.username;
   $scope.getMainCategories();
});

And in the second controller you'd just do

$scope.initRestId = function(){
   $scope.$broadcast("myEvent", {username: $scope.user.username });
};

Edit: Realised it was communication between two modules

Can you try including the firstApp module as a dependency to the secondApp where you declare the angular.module. That way you can communicate to the other app.

GROUP BY and COUNT in PostgreSQL

Using OVER() and LIMIT 1:

SELECT COUNT(1) OVER()
FROM posts 
   INNER JOIN votes ON votes.post_id = posts.id 
GROUP BY posts.id
LIMIT 1;

How to get all of the immediate subdirectories in Python

I have to mention the path.py library, which I use very often.

Fetching the immediate subdirectories become as simple as that:

my_dir.dirs()

The full working example is:

from path import Path

my_directory = Path("path/to/my/directory")

subdirs = my_directory.dirs()

NB: my_directory still can be manipulated as a string, since Path is a subclass of string, but providing a bunch of useful methods for manipulating paths

How to cat <<EOF >> a file containing code?

I know this is a two year old question, but this is a quick answer for those searching for a 'how to'.

If you don't want to have to put quotes around anything you can simply write a block of text to a file, and escape variables you want to export as text (for instance for use in a script) and not escape one's you want to export as the value of the variable.

#!/bin/bash

FILE_NAME="test.txt"
VAR_EXAMPLE="\"string\""

cat > ${FILE_NAME} << EOF
\${VAR_EXAMPLE}=${VAR_EXAMPLE} in ${FILE_NAME}  
EOF

Will write "${VAR_EXAMPLE}="string" in test.txt" into test.txt

This can also be used to output blocks of text to the console with the same rules by omitting the file name

#!/bin/bash

VAR_EXAMPLE="\"string\""

cat << EOF
\${VAR_EXAMPLE}=${VAR_EXAMPLE} to console 
EOF

Will output "${VAR_EXAMPLE}="string" to console" to the console

Easier way to create circle div than using an image?

It is actually possible.

See: CSS Tip: How to Make Circles Without Images. See demo.

But be warned, It has serious disadvantages in terms of compatibility basically, you are making a cat bark.

See it working here

As you will see you just have to set up the height and width to half the border-radius

Good luck!

how to call a function from another function in Jquery

I assume you don't want to rebind the event, but call the handler.

You can use trigger() to trigger events:

$('#billing_state_id').trigger('change');

If your handler doesn't rely on the event context and you don't want to trigger other handlers for the event, you could also name the function:

function someFunction() {
    //do stuff
}

$(document).ready(function(){
    //Load City by State
    $('#billing_state_id').live('change', someFunction);   
    $('#click_me').live('click', function() {
       //do something
       someFunction();
    });
  });

Also note that live() is deprecated, on() is the new hotness.

How to change CSS using jQuery?

_x000D_
_x000D_
$(function(){ _x000D_
$('.bordered').css({_x000D_
"border":"1px solid #EFEFEF",_x000D_
"margin":"0 auto",_x000D_
"width":"80%"_x000D_
});_x000D_
_x000D_
$('h1').css({_x000D_
"margin-left":"10px"_x000D_
});_x000D_
_x000D_
$('#myParagraph').css({_x000D_
"margin-left":"10px",_x000D_
"font-family":"sans-serif"_x000D_
});_x000D_
_x000D_
});
_x000D_
<script src="https://ajax.googleapis.com/ajax/libs/jquery/2.1.0/jquery.min.js"></script>_x000D_
<div class="bordered">_x000D_
<h1>Header</h1>_x000D_
<p id="myParagraph">This is some paragraph text</p>_x000D_
</div>
_x000D_
_x000D_
_x000D_

Regex for quoted string with escaping quotes

Most of the solutions provided here use alternative repetition paths i.e. (A|B)*.

You may encounter stack overflows on large inputs since some pattern compiler implements this using recursion.

Java for instance: http://bugs.java.com/bugdatabase/view_bug.do?bug_id=6337993

Something like this: "(?:[^"\\]*(?:\\.)?)*", or the one provided by Guy Bedford will reduce the amount of parsing steps avoiding most stack overflows.

How to read a text file into a list or an array with Python

You will have to split your string into a list of values using split()

So,

lines = text_file.read().split(',')

EDIT: I didn't realise there would be so much traction to this. Here's a more idiomatic approach.

import csv
with open('filename.csv', 'r') as fd:
    reader = csv.reader(fd)
    for row in reader:
        # do something

What is SELF JOIN and when would you use it?

Well, one classic example is where you wanted to get a list of employees and their immediate managers:

select e.employee as employee, b.employee as boss
from emptable e, emptable b
where e.manager_id = b.empolyee_id
order by 1

It's basically used where there is any relationship between rows stored in the same table.

  • employees.
  • multi-level marketing.
  • machine parts.

And so on...

What is /var/www/html?

In the most shared hosts you can't set it.

On a VPS or dedicated server, you can set it, but everything has its price.

On shared hosts, in general you receive a Linux account, something such as /home/(your username)/, and the equivalent of /var/www/html turns to /home/(your username)/public_html/ (or something similar, such as /home/(your username)/www)

If you're accessing your account via FTP, you automatically has accessing the your */home/(your username)/ folder, just find the www or public_html and put your site in it.

If you're using absolute path in the code, bad news, you need to refactor it to use relative paths in the code, at least in a shared host.

Oracle client and networking components were not found

After you install Oracle Client components on the remote server, restart SQL Server Agent from the PC Management Console or directly from Sql Server Management Studio. This will allow the service to load correctly the path to the Oracle components. Otherwise your package will work on design time but fail on run time.

Java equivalent to C# extension methods

Bit late to the party on this question, but in case anyone finds it useful I just created a subclass:

public class ArrayList2<T> extends ArrayList<T> 
{
    private static final long serialVersionUID = 1L;

    public T getLast()
    {
        if (this.isEmpty())
        {
            return null;
        }
        else
        {       
            return this.get(this.size() - 1);
        }
    }
}

How to create query parameters in Javascript?

Zabba has provided in a comment on the currently accepted answer a suggestion that to me is the best solution: use jQuery.param().

If I use jQuery.param() on the data in the original question, then the code is simply:

const params = jQuery.param({
    var1: 'value',
    var2: 'value'
});

The variable params will be

"var1=value&var2=value"

For more complicated examples, inputs and outputs, see the jQuery.param() documentation.

Add single element to array in numpy

append() creates a new array which can be the old array with the appended element.

I think it's more normal to use the proper method for adding an element:

a = numpy.append(a, a[0])

How to pip or easy_install tkinter on Windows

if your using python 3.4.1 just write this line from tkinter import * this will put everything in the module into the default namespace of your program. in fact instead of referring to say a button like tkinter.Button you just type Button

How do I detect a page refresh using jquery?

if you want to bookkeep some variable before page refresh

$(window).on('beforeunload', function(){
    // your logic here
});

if you want o load some content base on some condition

$(window).on('load', function(){
    // your logic here`enter code here`
});

HikariCP - connection is not available

From stack trace:

HikariPool: Timeout failure pool HikariPool-0 stats (total=20, active=20, idle=0, waiting=0) Means pool reached maximum connections limit set in configuration.

The next line: HikariPool-0 - Connection is not available, request timed out after 30000ms. Means pool waited 30000ms for free connection but your application not returned any connection meanwhile.

Mostly it is connection leak (connection is not closed after borrowing from pool), set leakDetectionThreshold to the maximum value that you expect SQL query would take to execute.

otherwise, your maximum connections 'at a time' requirement is higher than 20 !

How to retrieve all keys (or values) from a std::map and put them into a vector?

The best non-sgi, non-boost STL solution is to extend map::iterator like so:

template<class map_type>
class key_iterator : public map_type::iterator
{
public:
    typedef typename map_type::iterator map_iterator;
    typedef typename map_iterator::value_type::first_type key_type;

    key_iterator(const map_iterator& other) : map_type::iterator(other) {} ;

    key_type& operator *()
    {
        return map_type::iterator::operator*().first;
    }
};

// helpers to create iterators easier:
template<class map_type>
key_iterator<map_type> key_begin(map_type& m)
{
    return key_iterator<map_type>(m.begin());
}
template<class map_type>
key_iterator<map_type> key_end(map_type& m)
{
    return key_iterator<map_type>(m.end());
}

and then use them like so:

        map<string,int> test;
        test["one"] = 1;
        test["two"] = 2;

        vector<string> keys;

//      // method one
//      key_iterator<map<string,int> > kb(test.begin());
//      key_iterator<map<string,int> > ke(test.end());
//      keys.insert(keys.begin(), kb, ke);

//      // method two
//      keys.insert(keys.begin(),
//           key_iterator<map<string,int> >(test.begin()),
//           key_iterator<map<string,int> >(test.end()));

        // method three (with helpers)
        keys.insert(keys.begin(), key_begin(test), key_end(test));

        string one = keys[0];

How can I find all matches to a regular expression in Python?

Use re.findall or re.finditer instead.

re.findall(pattern, string) returns a list of matching strings.

re.finditer(pattern, string) returns an iterator over MatchObject objects.

Example:

re.findall( r'all (.*?) are', 'all cats are smarter than dogs, all dogs are dumber than cats')
# Output: ['cats', 'dogs']

[x.group() for x in re.finditer( r'all (.*?) are', 'all cats are smarter than dogs, all dogs are dumber than cats')]
# Output: ['all cats are', 'all dogs are']

Get Bitmap attached to ImageView

Bitmap bitmap = ((BitmapDrawable)image.getDrawable()).getBitmap();

How to Specify "Vary: Accept-Encoding" header in .htaccess

I guess it's meant that you enable gzip compression for your css and js files, because that will enable the client to receive both gzip-encoded content and a plain content.

This is how to do it in apache2:

<IfModule mod_deflate.c>
    #The following line is enough for .js and .css
    AddOutputFilter DEFLATE js css

    #The following line also enables compression by file content type, for the following list of Content-Type:s
    AddOutputFilterByType DEFLATE text/html text/plain text/xml application/xml

    #The following lines are to avoid bugs with some browsers
    BrowserMatch ^Mozilla/4 gzip-only-text/html
    BrowserMatch ^Mozilla/4\.0[678] no-gzip
    BrowserMatch \bMSIE !no-gzip !gzip-only-text/html 
</IfModule>

And here's how to add the Vary Accept-Encoding header: [src]

<IfModule mod_headers.c>
  <FilesMatch "\.(js|css|xml|gz)$">
    Header append Vary: Accept-Encoding
  </FilesMatch>
</IfModule>

The Vary: header tells the that the content served for this url will vary according to the value of a certain request header. Here it says that it will serve different content for clients who say they Accept-Encoding: gzip, deflate (a request header), than the content served to clients that do not send this header. The main advantage of this, AFAIK, is to let intermediate caching proxies know they need to have two different versions of the same url because of such change.

How to switch databases in psql?

Listing and Switching Databases in PostgreSQL When you need to change between databases, you’ll use the \connect command, or \c followed by the database name as shown below:

postgres=# \connect database_name
postgres=# \c database_name

Check the database you are currently connected to.

SELECT current_database();

PostgreSQL List Databases

postgres=# \l
 postgres=# \list

Django 1.7 throws django.core.exceptions.AppRegistryNotReady: Models aren't loaded yet

The issue is in your registration app. It seems django-registration calls get_user_module() in models.py at a module level (when models are still being loaded by the application registration process). This will no longer work:

try:
    from django.contrib.auth import get_user_model
    User = get_user_model()
except ImportError:
    from django.contrib.auth.models import User    

I'd change this models file to only call get_user_model() inside methods (and not at module level) and in FKs use something like:

user = ForeignKey(settings.AUTH_USER_MODEL)

BTW, the call to django.setup() shouldn't be required in your manage.py file, it's called for you in execute_from_command_line. (source)

How to use "/" (directory separator) in both Linux and Windows in Python?

os.path.normpath(pathname) should also be mentioned as it converts / path separators into \ separators on Windows. It also collapses redundant uplevel references... i.e., A/B and A/foo/../B and A/./B all become A/B. And if you are Windows, these all become A\B.

Is it possible to set a timeout for an SQL query on Microsoft SQL server?

As far as I know, apart from setting the command or connection timeouts in the client, there is no way to change timeouts on a query by query basis in the server.

You can indeed change the default 600 seconds using sp_configure, but these are server scoped.

Changing the space between each item in Bootstrap navbar

As of Bootstrap 4, you can use the spacing utilities.

Add for instance px-2 in the classes of the nav-item to increase the padding.

Type safety: Unchecked cast

The solution to avoid the unchecked warning:

class MyMap extends HashMap<String, String> {};
someMap = (MyMap)getApplicationContext().getBean("someMap");

CORS jQuery AJAX request

It's easy, you should set server http response header first. The problem is not with your front-end javascript code. You need to return this header:

Access-Control-Allow-Origin:*

or

Access-Control-Allow-Origin:your domain

In Apache config files, the code is like this:

Header set Access-Control-Allow-Origin "*"

In nodejs,the code is like this:

res.setHeader('Access-Control-Allow-Origin','*');

How do I move a redis database from one server to another?

you can also use rdd

it can dump & restore a running redis server and allow filter/match/rename dumps keys

jQuery ui dialog change title after load-callback

Even better!

    jQuery( "#dialog" ).attr('title', 'Error');
    jQuery( "#dialog" ).text('You forgot to enter your first name');

How to convert a unix timestamp (seconds since epoch) to Ruby DateTime?

Time Zone Handling

I just want to clarify, even though this has been commented so future people don't miss this very important distinction.

DateTime.strptime("1318996912",'%s') # => Wed, 19 Oct 2011 04:01:52 +0000

displays a return value in UTC and requires the seconds to be a String and outputs a UTC Time object, whereas

Time.at(1318996912) # => 2011-10-19 00:01:52 -0400

displays a return value in the LOCAL time zone, normally requires a FixNum argument, but the Time object itself is still in UTC even though the display is not.

So even though I passed the same integer to both methods, I seemingly two different results because of how the class' #to_s method works. However, as @Eero had to remind me twice of:

Time.at(1318996912) == DateTime.strptime("1318996912",'%s') # => true

An equality comparison between the two return values still returns true. Again, this is because the values are basically the same (although different classes, the #== method takes care of this for you), but the #to_s method prints drastically different strings. Although, if we look at the strings, we can see they are indeed the same time, just printed in different time zones.

Method Argument Clarification

The docs also say "If a numeric argument is given, the result is in local time." which makes sense, but was a little confusing to me because they don't give any examples of non-integer arguments in the docs. So, for some non-integer argument examples:

Time.at("1318996912")
TypeError: can't convert String into an exact number

you can't use a String argument, but you can use a Time argument into Time.at and it will return the result in the time zone of the argument:

Time.at(Time.new(2007,11,1,15,25,0, "+09:00"))
=> 2007-11-01 15:25:00 +0900

Benchmarks

After a discussion with @AdamEberlin on his answer, I decided to publish slightly changed benchmarks to make everything as equal as possible. Also, I never want to have to build these again so this is as good a place as any to save them.

Time.at(int).to_datetime ~ 2.8x faster

09:10:58-watsw018:~$ ruby -v
ruby 2.3.7p456 (2018-03-28 revision 63024) [universal.x86_64-darwin18]
09:11:00-watsw018:~$ irb
irb(main):001:0> require 'benchmark'
=> true
irb(main):002:0> require 'date'
=> true
irb(main):003:0>
irb(main):004:0* format = '%s'
=> "%s"
irb(main):005:0> times = ['1318996912', '1318496913']
=> ["1318996912", "1318496913"]
irb(main):006:0> int_times = times.map(&:to_i)
=> [1318996912, 1318496913]
irb(main):007:0>
irb(main):008:0* datetime_from_strptime = DateTime.strptime(times.first, format)
=> #<DateTime: 2011-10-19T04:01:52+00:00 ((2455854j,14512s,0n),+0s,2299161j)>
irb(main):009:0> datetime_from_time = Time.at(int_times.first).to_datetime
=> #<DateTime: 2011-10-19T00:01:52-04:00 ((2455854j,14512s,0n),-14400s,2299161j)>
irb(main):010:0>
irb(main):011:0* datetime_from_strptime === datetime_from_time
=> true
irb(main):012:0>
irb(main):013:0* Benchmark.measure do
irb(main):014:1*   100_000.times {
irb(main):015:2*     times.each do |i|
irb(main):016:3*       DateTime.strptime(i, format)
irb(main):017:3>     end
irb(main):018:2>   }
irb(main):019:1> end
=> #<Benchmark::Tms:0x00007fbdc18f0d28 @label="", @real=0.8680500000045868, @cstime=0.0, @cutime=0.0, @stime=0.009999999999999998, @utime=0.86, @total=0.87>
irb(main):020:0>
irb(main):021:0* Benchmark.measure do
irb(main):022:1*   100_000.times {
irb(main):023:2*     int_times.each do |i|
irb(main):024:3*       Time.at(i).to_datetime
irb(main):025:3>     end
irb(main):026:2>   }
irb(main):027:1> end
=> #<Benchmark::Tms:0x00007fbdc3108be0 @label="", @real=0.33059399999910966, @cstime=0.0, @cutime=0.0, @stime=0.0, @utime=0.32000000000000006, @total=0.32000000000000006>

****edited to not be completely and totally incorrect in every way****

****added benchmarks****

Getting value from table cell in JavaScript...not jQuery

function GetCellValues() {
    var table = document.getElementById('mytable');
    for (var r = 0, n = table.rows.length; r < n; r++) {
        for (var c = 0, m = table.rows[r].cells.length; c < m; c++) {
            alert(table.rows[r].cells[c].innerHTML);
        }
    }
}

Accessing JPEG EXIF rotation data in JavaScript on the client side

I upload expansion code to show photo by android camera on html as normal on some img tag with right rotaion, especially for img tag whose width is wider than height. I know this code is ugly but you don't need to install any other packages. (I used above code to obtain exif rotation value, Thank you.)

function getOrientation(file, callback) {
  var reader = new FileReader();
  reader.onload = function(e) {

    var view = new DataView(e.target.result);
    if (view.getUint16(0, false) != 0xFFD8) return callback(-2);
    var length = view.byteLength, offset = 2;
    while (offset < length) {
      var marker = view.getUint16(offset, false);
      offset += 2;
      if (marker == 0xFFE1) {
        if (view.getUint32(offset += 2, false) != 0x45786966) return callback(-1);
        var little = view.getUint16(offset += 6, false) == 0x4949;
        offset += view.getUint32(offset + 4, little);
        var tags = view.getUint16(offset, little);
        offset += 2;
        for (var i = 0; i < tags; i++)
          if (view.getUint16(offset + (i * 12), little) == 0x0112)
            return callback(view.getUint16(offset + (i * 12) + 8, little));
      }
      else if ((marker & 0xFF00) != 0xFF00) break;
      else offset += view.getUint16(offset, false);
    }
    return callback(-1);
  };
  reader.readAsArrayBuffer(file);
}

var isChanged = false;
function rotate(elem, orientation) {
    if (isIPhone()) return;

    var degree = 0;
    switch (orientation) {
        case 1:
            degree = 0;
            break;
        case 2:
            degree = 0;
            break;
        case 3:
            degree = 180;
            break;
        case 4:
            degree = 180;
            break;
        case 5:
            degree = 90;
            break;
        case 6:
            degree = 90;
            break;
        case 7:
            degree = 270;
            break;
        case 8:
            degree = 270;
            break;
    }
    $(elem).css('transform', 'rotate('+ degree +'deg)')
    if(degree == 90 || degree == 270) {
        if (!isChanged) {
            changeWidthAndHeight(elem)
            isChanged = true
        }
    } else if ($(elem).css('height') > $(elem).css('width')) {
        if (!isChanged) {
            changeWidthAndHeightWithOutMargin(elem)
            isChanged = true
        } else if(degree == 180 || degree == 0) {
            changeWidthAndHeightWithOutMargin(elem)
            if (!isChanged)
                isChanged = true
            else
                isChanged = false
        }
    }
}


function changeWidthAndHeight(elem){
    var e = $(elem)
    var width = e.css('width')
    var height = e.css('height')
    e.css('width', height)
    e.css('height', width)
    e.css('margin-top', ((getPxInt(height) - getPxInt(width))/2).toString() + 'px')
    e.css('margin-left', ((getPxInt(width) - getPxInt(height))/2).toString() + 'px')
}

function changeWidthAndHeightWithOutMargin(elem){
    var e = $(elem)
    var width = e.css('width')
    var height = e.css('height')
    e.css('width', height)
    e.css('height', width)
    e.css('margin-top', '0')
    e.css('margin-left', '0')
}

function getPxInt(pxValue) {
    return parseInt(pxValue.trim("px"))
}

function isIPhone(){
    return (
        (navigator.platform.indexOf("iPhone") != -1) ||
        (navigator.platform.indexOf("iPod") != -1)
    );
}

and then use such as

$("#banner-img").change(function () {
    var reader = new FileReader();
    getOrientation(this.files[0], function(orientation) {
        rotate($('#banner-img-preview'), orientation, 1)
    });

    reader.onload = function (e) {
        $('#banner-img-preview').attr('src', e.target.result)
        $('#banner-img-preview').css('display', 'inherit')

    };

    // read the image file as a data URL.
    reader.readAsDataURL(this.files[0]);

});

Using Jasmine to spy on a function without an object

A very simple way:

import * as myFunctionContainer from 'whatever-lib';

const fooSpy = spyOn(myFunctionContainer, 'myFunc');

Rails: call another controller action from a controller

You can use url_for to get the URL for a controller and action and then use redirect_to to go to that URL.

redirect_to url_for(:controller => :controller_name, :action => :action_name)

Java resource as file

Here is a bit of code from one of my applications... Let me know if it suits your needs. You can use this if you know the file you want to use.

URL defaultImage = ClassA.class.getResource("/packageA/subPackage/image-name.png");
File imageFile = new File(defaultImage.toURI());

Hope that helps.

Best way to initialize (empty) array in PHP

In ECMAScript implementations (for instance, ActionScript or JavaScript), Array() is a constructor function and [] is part of the array literal grammar. Both are optimized and executed in completely different ways, with the literal grammar not being dogged by the overhead of calling a function.

PHP, on the other hand, has language constructs that may look like functions but aren't treated as such. Even with PHP 5.4, which supports [] as an alternative, there is no difference in overhead because, as far as the compiler/parser is concerned, they are completely synonymous.

// Before 5.4, you could only write
$array = array(
    "foo" => "bar",
    "bar" => "foo",
);

// As of PHP 5.4, the following is synonymous with the above
$array = [
    "foo" => "bar",
    "bar" => "foo",
];

If you need to support older versions of PHP, use the former syntax. There's also an argument for readability but, being a long-time JS developer, the latter seems rather natural to me. I actually made the mistake of trying to initialise arrays using [] when I was first learning PHP.

This change to the language was originally proposed and rejected due to a majority vote against by core developers with the following reason:

This patch will not be accepted because slight majority of the core developers voted against. Though if you take a accumulated mean between core developers and userland votes seems to show the opposite it would be irresponsible to submit a patch witch is not supported or maintained in the long run.

However, it appears there was a change of heart leading up to 5.4, perhaps influenced by the implementations of support for popular databases like MongoDB (which use ECMAScript syntax).

Any way to generate ant build.xml file automatically from Eclipse?

I have been trying to do the same myself. What I found was that the "Export Ant Buildfile" gets kicked off in the org.eclipse.ant.internal.ui.datatransfer.AntBuildfileExportPage.java file. This resides in the org.eclipse.ant.ui plugin.

To view the source, use the Plug-in Development perspective and open the Plug-ins view. Then right-click on the org.eclipse.ant.ui plugin and select import as > source project.

My plan is to create a Java program to programmatically kick off the ant buildfile generation and call this in an Ant file every time I build by adding the ant file to the builders of my projects (Right-click preferences on a projet, under the builders tab).

How can I debug git/git-shell related problems?

For older git versions (1.8 and before)

I could find no suitable way to enable SSH debugging in an older git and ssh versions. I looked for environment variables using ltrace -e getenv ... and couldn't find any combination of GIT_TRACE or SSH_DEBUG variables that would work.

Instead here's a recipe to temporarily inject 'ssh -v' into the git->ssh sequence:

$ echo '/usr/bin/ssh -v ${@}' >/tmp/ssh
$ chmod +x /tmp/ssh
$ PATH=/tmp:${PATH} git clone ...
$ rm -f /tmp/ssh

Here's output from git version 1.8.3 with ssh version OpenSSH_5.3p1, OpenSSL 1.0.1e-fips 11 Feb 2013 cloning a github repo:

$ (echo '/usr/bin/ssh -v ${@}' >/tmp/ssh; chmod +x /tmp/ssh; PATH=/tmp:${PATH} \
   GIT_TRACE=1 git clone https://github.com/qneill/cliff.git; \
   rm -f /tmp/ssh) 2>&1 | tee log
trace: built-in: git 'clone' 'https://github.com/qneill/cliff.git'
trace: run_command: 'git-remote-https' 'origin' 'https://github.com/qneill/cliff.git'
Cloning into 'cliff'...
OpenSSH_5.3p1, OpenSSL 1.0.1e-fips 11 Feb 2013
debug1: Reading configuration data /home/q.neill/.ssh/config
debug1: Reading configuration data /etc/ssh/ssh_config
debug1: Applying options for *
debug1: Connecting to github.com ...
...
Transferred: sent 4120, received 724232 bytes, in 0.2 seconds
Bytes per second: sent 21590.6, received 3795287.2
debug1: Exit status 0
trace: run_command: 'rev-list' '--objects' '--stdin' '--not' '--all'
trace: exec: 'git' 'rev-list' '--objects' '--stdin' '--not' '--all'
trace: built-in: git 'rev-list' '--objects' '--stdin' '--not' '--all'

C# error: Use of unassigned local variable

The compiler only knows that the code is or isn't reachable if you use "return". Think of Environment.Exit() as a function that you call, and the compiler don't know that it will close the application.

Difference between volatile and synchronized in Java

synchronized is method level/block level access restriction modifier. It will make sure that one thread owns the lock for critical section. Only the thread,which own a lock can enter synchronized block. If other threads are trying to access this critical section, they have to wait till current owner releases the lock.

volatile is variable access modifier which forces all threads to get latest value of the variable from main memory. No locking is required to access volatile variables. All threads can access volatile variable value at same time.

A good example to use volatile variable : Date variable.

Assume that you have made Date variable volatile. All the threads, which access this variable always get latest data from main memory so that all threads show real (actual) Date value. You don't need different threads showing different time for same variable. All threads should show right Date value.

enter image description here

Have a look at this article for better understanding of volatile concept.

Lawrence Dol cleary explained your read-write-update query.

Regarding your other queries

When is it more suitable to declare variables volatile than access them through synchronized?

You have to use volatile if you think all threads should get actual value of the variable in real time like the example I have explained for Date variable.

Is it a good idea to use volatile for variables that depend on input?

Answer will be same as in first query.

Refer to this article for better understanding.

How do I get the collection of Model State Errors in ASP.NET MVC?

This will give you one string with all the errors with comma separating

string validationErrors = string.Join(",",
                    ModelState.Values.Where(E => E.Errors.Count > 0)
                    .SelectMany(E => E.Errors)
                    .Select(E => E.ErrorMessage)
                    .ToArray());

How to round up a number to nearest 10?

My first impulse was to google for "php math" and I discovered that there's a core math library function called "round()" that likely is what you want.

Auto-expanding layout with Qt-Designer

Set the horizontalPolicy & VerticalPolicy for the controls/widgets to "Preferred".

How do I handle too long index names in a Ruby on Rails ActiveRecord migration?

create_table :you_table_name do |t| t.references :studant, index: { name: 'name_for_studant_index' } t.references :teacher, index: { name: 'name_for_teacher_index' } end

How do I initialize the base (super) class?

As of python 3.5.2, you can use:

class C(B):
def method(self, arg):
    super().method(arg)    # This does the same thing as:
                           # super(C, self).method(arg)

https://docs.python.org/3/library/functions.html#super

"No X11 DISPLAY variable" - what does it mean?

Very Easy, Had this same problem then what i did was to download and install an app that would help in displaying then fixed the error.

Download this app xming:

http://sourceforge.net/project/downloading.php?

Install, then use settings on this link:

http://www.geo.mtu.edu/geoschem/docs/putty_install.html or follow this steps:

Installing/Configuring PuTTy and Xming

Once PuTTy and Xming have been downloaded to the PC, install according to their respective instructions.

Configuring Xming

Once Xming is installed, run the application called 'XLaunch' and verify that the settings are as shown:

  • select Default entries on Display Settings windows, click next
  • click next on Session Type window.
  • click next on Additional parameters window(Notice clipboard checkbox is true)
  • save configuration and click to finish.

Configuring PuTTy

After installing PuTTy, double-click on the PuTTy icon on the desktop and configure as shown:

This shows creating a login profile then saving it.

  • On ssh -> X11, click on checkbox to enable X11 forwarding.
  • on X display location textbox, type localhost:0.0

save profile then connect remotely to server to test.

Cheers!!!

Credit card payment gateway in PHP?

Braintree also has an open source PHP library that makes PHP integration pretty easy.

Backbone.js fetch with parameters

Another example if you are using Titanium Alloy:

 collection.fetch({ 
     data: {
             where : JSON.stringify({
                page: 1
             })
           } 
      });

bash: mkvirtualenv: command not found

Try:

source `which virtualenvwrapper.sh`

The backticks are command substitution - they take whatever the program prints out and put it in the expression. In this case "which" checks the $PATH to find virtualenvwrapper.sh and outputs the path to it. The script is then read by the shell via 'source'.

If you want this to happen every time you restart your shell, it's probably better to grab the output from the "which" command first, and then put the "source" line in your shell, something like this:

echo "source /path/to/virtualenvwrapper.sh" >> ~/.profile

^ This may differ slightly based on your shell. Also, be careful not to use the a single > as this will truncate your ~/.profile :-o

What's better at freeing memory with PHP: unset() or $var = null

It works in a different way for variables copied by reference:

$a = 5;
$b = &$a;
unset($b); // just say $b should not point to any variable
print $a; // 5

$a = 5;
$b = &$a;
$b = null; // rewrites value of $b (and $a)
print $a; // nothing, because $a = null

How to get length of a list of lists in python

This saves the data in a list of lists.

text = open("filetest.txt", "r")
data = [ ]
for line in text:
    data.append( line.strip().split() )

print "number of lines ", len(data)
print "number of columns ", len(data[0])

print "element in first row column two ", data[0][1]

Concatenation of strings in Lua

Concatenation:

The string concatenation operator in Lua is denoted by two dots ('..'). If both operands are strings or numbers, then they are converted to strings according to the rules mentioned in §2.2.1. Otherwise, the "concat" metamethod is called (see §2.8).

from: http://www.lua.org/manual/5.1/manual.html#2.5.4

How to change UIPickerView height

stockPicker = [[UIPickerView alloc] init];
stockPicker.frame = CGRectMake(70.0,155, 180,100);

If You want to set the size of UiPickerView. Above code is surely gonna work for u.

Check if a string has white space

Your regex won't match anything, as it is. You definitely need to remove the quotes -- the "/" characters are sufficient.

/^\s+$/ is checking whether the string is ALL whitespace:

  • ^ matches the start of the string.
  • \s+ means at least 1, possibly more, spaces.
  • $ matches the end of the string.

Try replacing the regex with /\s/ (and no quotes)

How to copy data from one HDFS to another HDFS?

DistCp (distributed copy) is a tool used for copying data between clusters. It uses MapReduce to effect its distribution, error handling and recovery, and reporting. It expands a list of files and directories into input to map tasks, each of which will copy a partition of the files specified in the source list.

Usage: $ hadoop distcp <src> <dst>

example: $ hadoop distcp hdfs://nn1:8020/file1 hdfs://nn2:8020/file2

file1 from nn1 is copied to nn2 with filename file2

Distcp is the best tool as of now. Sqoop is used to copy data from relational database to HDFS and vice versa, but not between HDFS to HDFS.

More info:

There are two versions available - runtime performance in distcp2 is more compared to distcp

How to use MySQLdb with Python and Django in OSX 10.6?

I overcame the same problem by installing MySQL-python library using pip. You can see the message displayed on my console when I first changed my database settings in settings.py and executed makemigrations command(The solution is following the below message, just see that).

  (vir_env) admins-MacBook-Pro-3:src admin$ python manage.py makemigrations
Traceback (most recent call last):
  File "manage.py", line 10, in <module>
    execute_from_command_line(sys.argv)
  File "/Users/admin/Desktop/SetUp1/vir_env/lib/python2.7/site-packages/django/core/management/__init__.py", line 338, in execute_from_command_line
    utility.execute()
  File "/Users/admin/Desktop/SetUp1/vir_env/lib/python2.7/site-packages/django/core/management/__init__.py", line 312, in execute
    django.setup()
  File "/Users/admin/Desktop/SetUp1/vir_env/lib/python2.7/site-packages/django/__init__.py", line 18, in setup
    apps.populate(settings.INSTALLED_APPS)
  File "/Users/admin/Desktop/SetUp1/vir_env/lib/python2.7/site-packages/django/apps/registry.py", line 108, in populate
    app_config.import_models(all_models)
  File "/Users/admin/Desktop/SetUp1/vir_env/lib/python2.7/site-packages/django/apps/config.py", line 198, in import_models
    self.models_module = import_module(models_module_name)
  File "/usr/local/Cellar/python/2.7.12_1/Frameworks/Python.framework/Versions/2.7/lib/python2.7/importlib/__init__.py", line 37, in import_module
    __import__(name)
  File "/Users/admin/Desktop/SetUp1/vir_env/lib/python2.7/site-packages/django/contrib/auth/models.py", line 41, in <module>
    class Permission(models.Model):
  File "/Users/admin/Desktop/SetUp1/vir_env/lib/python2.7/site-packages/django/db/models/base.py", line 139, in __new__
    new_class.add_to_class('_meta', Options(meta, **kwargs))
  File "/Users/admin/Desktop/SetUp1/vir_env/lib/python2.7/site-packages/django/db/models/base.py", line 324, in add_to_class
    value.contribute_to_class(cls, name)
  File "/Users/admin/Desktop/SetUp1/vir_env/lib/python2.7/site-packages/django/db/models/options.py", line 250, in contribute_to_class
    self.db_table = truncate_name(self.db_table, connection.ops.max_name_length())
  File "/Users/admin/Desktop/SetUp1/vir_env/lib/python2.7/site-packages/django/db/__init__.py", line 36, in __getattr__
    return getattr(connections[DEFAULT_DB_ALIAS], item)
  File "/Users/admin/Desktop/SetUp1/vir_env/lib/python2.7/site-packages/django/db/utils.py", line 240, in __getitem__
    backend = load_backend(db['ENGINE'])
  File "/Users/admin/Desktop/SetUp1/vir_env/lib/python2.7/site-packages/django/db/utils.py", line 111, in load_backend
    return import_module('%s.base' % backend_name)
  File "/usr/local/Cellar/python/2.7.12_1/Frameworks/Python.framework/Versions/2.7/lib/python2.7/importlib/__init__.py", line 37, in import_module
    __import__(name)
  File "/Users/admin/Desktop/SetUp1/vir_env/lib/python2.7/site-packages/django/db/backends/mysql/base.py", line 27, in <module>
    raise ImproperlyConfigured("Error loading MySQLdb module: %s" % e)
django.core.exceptions.ImproperlyConfigured: Error loading MySQLdb module: No module named MySQLdb

Finally I overcame this problem as follows:

(vir_env) admins-MacBook-Pro-3:src admin$ pip install MySQLdb
Collecting MySQLdb
  Could not find a version that satisfies the requirement MySQLdb (from versions: )
No matching distribution found for MySQLdb
(vir_env) admins-MacBook-Pro-3:src admin$ pip install MySQL-python
Collecting MySQL-python
  Downloading MySQL-python-1.2.5.zip (108kB)
    100% |¦¦¦¦¦¦¦¦¦¦¦¦¦¦¦¦¦¦¦¦¦¦¦¦¦¦¦¦¦¦¦¦| 112kB 364kB/s 
Building wheels for collected packages: MySQL-python
  Running setup.py bdist_wheel for MySQL-python ... done
  Stored in directory: /Users/admin/Library/Caches/pip/wheels/38/a3/89/ec87e092cfb38450fc91a62562055231deb0049a029054dc62
Successfully built MySQL-python
Installing collected packages: MySQL-python
Successfully installed MySQL-python-1.2.5
(vir_env) admins-MacBook-Pro-3:src admin$ python manage.py makemigrations
No changes detected
(vir_env) admins-MacBook-Pro-3:src admin$ python manage.py migrate
Operations to perform:
  Synchronize unmigrated apps: staticfiles, rest_framework, messages, crispy_forms
  Apply all migrations: admin, contenttypes, sessions, auth, PyApp
Synchronizing apps without migrations:
  Creating tables...
    Running deferred SQL...
  Installing custom SQL...
Running migrations:
  Rendering model states... DONE
  Applying PyApp.0001_initial... OK
  Applying PyApp.0002_auto_20170310_0936... OK
  Applying PyApp.0003_auto_20170310_0953... OK
  Applying PyApp.0004_auto_20170310_0954... OK
  Applying PyApp.0005_auto_20170311_0619... OK
  Applying PyApp.0006_auto_20170311_0622... OK
  Applying PyApp.0007_loraevksensor... OK
  Applying PyApp.0008_auto_20170315_0752... OK
  Applying PyApp.0009_auto_20170315_0753... OK
  Applying PyApp.0010_auto_20170315_0806... OK
  Applying PyApp.0011_auto_20170315_0814... OK
  Applying PyApp.0012_auto_20170315_0820... OK
  Applying PyApp.0013_auto_20170315_0822... OK
  Applying PyApp.0014_auto_20170315_0907... OK
  Applying PyApp.0015_auto_20170315_1041... OK
  Applying PyApp.0016_auto_20170315_1355... OK
  Applying PyApp.0017_auto_20170315_1401... OK
  Applying PyApp.0018_auto_20170331_1348... OK
  Applying PyApp.0019_auto_20170331_1349... OK
  Applying PyApp.0020_auto_20170331_1350... OK
  Applying PyApp.0021_auto_20170331_1458... OK
  Applying PyApp.0022_delete_postoffice... OK
  Applying PyApp.0023_posoffice... OK
  Applying PyApp.0024_auto_20170331_1504... OK
  Applying PyApp.0025_auto_20170331_1511... OK
  Applying contenttypes.0001_initial... OK
  Applying auth.0001_initial... OK
  Applying admin.0001_initial... OK
  Applying contenttypes.0002_remove_content_type_name... OK
  Applying auth.0002_alter_permission_name_max_length... OK
  Applying auth.0003_alter_user_email_max_length... OK
  Applying auth.0004_alter_user_username_opts... OK
  Applying auth.0005_alter_user_last_login_null... OK
  Applying auth.0006_require_contenttypes_0002... OK
  Applying sessions.0001_initial... OK
(vir_env) admins-MacBook-Pro-3:src admin$ 

how to instanceof List<MyType>?

    if (list instanceof List && ((List) list).stream()
                                             .noneMatch((o -> !(o instanceof MyType)))) {}

New features in java 7

The following list contains links to the the enhancements pages in the Java SE 7.

Swing
IO and New IO
Networking
Security
Concurrency Utilities
Rich Internet Applications (RIA)/Deployment
    Requesting and Customizing Applet Decoration in Dragg able Applets
    Embedding JNLP File in Applet Tag
    Deploying without Codebase
    Handling Applet Initialization Status with Event Handlers
Java 2D
Java XML – JAXP, JAXB, and JAX-WS
Internationalization
java.lang Package
    Multithreaded Custom Class Loaders in Java SE 7
Java Programming Language
    Binary Literals
    Strings in switch Statements
    The try-with-resources Statement
    Catching Multiple Exception Types and Rethrowing Exceptions with Improved Type Checking
    Underscores in Numeric Literals
    Type Inference for Generic Instance Creation
    Improved Compiler Warnings and Errors When Using Non-Reifiable Formal Parameters with Varargs Methods
Java Virtual Machine (JVM)
    Java Virtual Machine Support for Non-Java Languages
    Garbage-First Collector
    Java HotSpot Virtual Machine Performance Enhancements
JDBC

Reference 1 Reference 2

How to use RANK() in SQL Server

RANK() is good, but it assigns the same rank for equal or similar values. And if you need unique rank, then ROW_NUMBER() solves this problem

ROW_NUMBER() OVER (ORDER BY totals DESC) AS xRank

Store an array in HashMap

If you want to store multiple values for a key (if I understand you correctly), you could try a MultiHashMap (available in various libraries, not only commons-collections).

Beautiful Soup and extracting a div and its contents by ID

have you tried soup.findAll("div", {"id": "articlebody"})?

sounds crazy, but if you're scraping stuff from the wild, you can't rule out multiple divs...

Markdown and including multiple files

The short answer is no. The long answer is yes. :-)

Markdown was designed to allow people to write simple, readable text that could be easily converted to a simple HTML markup. It doesn't really do document layout. For example, there's no real way to align an image to the right or left. As to your question, there's no markdown command to include a single link from one file to another in any version of markdown (so far as I know).

The closest you could come to this functionality is Pandoc. Pandoc allows you to merge files as a part of the transformation, which allows you to easily render multiple files into a single output. For example, if you were creating a book, then you could have chapters like this:

01_preface.md
02_introduction.md
03_why_markdown_is_useful.md
04_limitations_of_markdown.md
05_conclusions.md

You can merge them by doing executing this command within the same directory:

pandoc *.md > markdown_book.html

Since pandoc will merge all the files prior to doing the translation, you can include your links in the last file like this:

01_preface.md
02_introduction.md
03_why_markdown_is_useful.md
04_limitations_of_markdown.md
05_conclusions.md
06_links.md

So part of your 01_preface.md could look like this:

I always wanted to write a book with [markdown][mkdnlink].

And part of your 02_introduction.md could look like this:

Let's start digging into [the best text-based syntax][mkdnlink] available.

As long as your last file includes the line:

[mkdnlink]: http://daringfireball.net/projects/markdown

...the same command used before will perform the merge and conversion while including that link throughout. Just make sure you leave a blank line or two at the beginning of that file. The pandoc documentation says that it adds a blank line between files that are merged this way, but this didn't work for me without the blank line.

How do I create a simple 'Hello World' module in Magento?

I've been wrestling with Magento for the last month or so and I'm still trying to figure it out. So this is a case of the blind leading the blind. There's little in the way of documentation and the forum/wiki is chaotic at best. Not only that, but there are several solutions that are either outdated or far from optimal. I'm not sure if you have a project or just trying to figure it out, but it's probably easier if you started with modifying existing functionality as opposed to creating something completely new. For that I'd definately go with the "Recommended articles for developers" in the wiki. The new payment method one was a real eye-opener.

For debugging I'd definitely recommend using FirePHP and looking at your HTML source when something goes wrong. The ole echo debug method doesn't really work all that well.

The general architecture is so mind-numbingly complex, that even if I completely understood it, I'd need to write a book to cover it. The best I can do is give you advice I wish someone had given me when I first started...

Stay away from core files. Don't modify them, instead write your own module and override what you need.

Magento uses config files consisting of XML to decide what it needs to do. In order to get it to run your own stuff as opposed to core functionality you need the correct xml. Unfortunately there is no guide on how to build you XML; you need to look at examples and do some serious testing. To complicate things the content of these files is largely case-sensitive. However if you master these you can override any part of the basic functionality which makes for a very powerful system.

Magento uses methods like Mage::getModel('mymodel'), Mage::getSingleton('mysingleton'), Mage::helper('myhelper') to return objects of certain classes. It finds these by default in its core namespace. If you want it to use your own, you need to override these in your config.xml file.

The name of your classes must correspond to the folder they're in.

A lot of the objects in Magento ultimately extend something called a Varien_Object. This is a general purpose class (kind of like a swiss army knife) and its purpose in life is to allow you to define your own methods/variables on the fly. For example you'll see it used as a glorified array to pass data from one method to another.

During development make sure you caching is disabled. It'll make magento excruciatingly slow, but it'll save you a lot of head trauma (from banging it on your desk).

You'll see $this being used a lot. It means a different class depending on what file you see it. get_class($this) is your friend, especially in conjunction with FirePHP.

Jot things down on paper. A lot. There are countless little factoids that you're gonna need 1-2 days after you encounter them.

Magento loves OO. Don't be surprised if tracing a method takes you through 5-10 different classes.

Read the designer's guide here. It's meant mostly for graphics designers, but you need it to understand where and why the output from your module will end up. For that don't forget to turn on "Template path hints" in the developer section of the admin panel.

There's more, but I'll stop here before this turns into a dissertation.

Java, how to compare Strings with String Arrays

import java.util.Scanner;
import java.util.*;
public class Main
{
  public static void main (String[]args) throws Exception
  {
    Scanner in = new Scanner (System.in);
    /*Prints out the welcome message at the top of the screen */
      System.out.printf ("%55s", "**WELCOME TO IDIOCY CENTRAL**\n");
      System.out.printf ("%55s", "=================================\n");

      String[] codes =
    {
    "G22", "K13", "I30", "S20"};

      System.out.printf ("%5s%5s%5s%5s\n", codes[0], codes[1], codes[2],
             codes[3]);
      System.out.printf ("Enter one of the above!\n");

    String usercode = in.nextLine ();
    for (int i = 0; i < codes.length; i++)
      {
    if (codes[i].equals (usercode))
      {
        System.out.printf ("What's the matter with you?\n");
      }
    else
      {
        System.out.printf ("Youda man!");
      }
      }

  }
}

Track a new remote branch created on GitHub

If you don't have an existing local branch, it is truly as simple as:

git fetch
git checkout <remote-branch-name>

For instance if you fetch and there is a new remote tracking branch called origin/feature/Main_Page, just do this:

git checkout feature/Main_Page

This creates a local branch with the same name as the remote branch, tracking that remote branch. If you have multiple remotes with the same branch name, you can use the less ambiguous:

git checkout -t <remote>/<remote-branch-name>

If you already made the local branch and don't want to delete it, see How do you make an existing Git branch track a remote branch?.

Copy a file in a sane, safe and efficient way

I'm not quite sure what a "good way" of copying a file is, but assuming "good" means "fast", I could broaden the subject a little.

Current operating systems have long been optimized to deal with run of the mill file copy. No clever bit of code will beat that. It is possible that some variant of your copy techniques will prove faster in some test scenario, but they most likely would fare worse in other cases.

Typically, the sendfile function probably returns before the write has been committed, thus giving the impression of being faster than the rest. I haven't read the code, but it is most certainly because it allocates its own dedicated buffer, trading memory for time. And the reason why it won't work for files bigger than 2Gb.

As long as you're dealing with a small number of files, everything occurs inside various buffers (the C++ runtime's first if you use iostream, the OS internal ones, apparently a file-sized extra buffer in the case of sendfile). Actual storage media is only accessed once enough data has been moved around to be worth the trouble of spinning a hard disk.

I suppose you could slightly improve performances in specific cases. Off the top of my head:

  • If you're copying a huge file on the same disk, using a buffer bigger than the OS's might improve things a bit (but we're probably talking about gigabytes here).
  • If you want to copy the same file on two different physical destinations you will probably be faster opening the three files at once than calling two copy_file sequentially (though you'll hardly notice the difference as long as the file fits in the OS cache)
  • If you're dealing with lots of tiny files on an HDD you might want to read them in batches to minimize seeking time (though the OS already caches directory entries to avoid seeking like crazy and tiny files will likely reduce disk bandwidth dramatically anyway).

But all that is outside the scope of a general purpose file copy function.

So in my arguably seasoned programmer's opinion, a C++ file copy should just use the C++17 file_copy dedicated function, unless more is known about the context where the file copy occurs and some clever strategies can be devised to outsmart the OS.

does linux shell support list data structure?

For make a list, simply do that

colors=(red orange white "light gray")

Technically is an array, but - of course - it has all list features.
Even python list are implemented with array

Encoding as Base64 in Java

In Java 7 I coded this method

import javax.xml.bind.DatatypeConverter;

public static String toBase64(String data) {
    return DatatypeConverter.printBase64Binary(data.getBytes());
}

Automatically start forever (node) on system restart

Copied answer from the attached question.

You can use PM2, it's a production process manager for Node.js applications with a built-in load balancer.

Install PM2

$ npm install pm2 -g

Start an application

$ pm2 start app.js

If you using express then you can start your app like

pm2 start ./bin/www --name="app"

Listing all running processes:

$ pm2 list

It will list all process. You can then stop / restart your service by using ID or Name of the app with following command.

$ pm2 stop all                  
$ pm2 stop 0                    
$ pm2 restart all               

To display logs

$ pm2 logs ['all'|app_name|app_id]

Setting the default value of a DateTime Property to DateTime.Now inside the System.ComponentModel Default Value Attrbute

In C# Version 6 it's possible to provide a default value

public DateTime fieldname { get; set; } = DateTime.Now;

No Spring WebApplicationInitializer types detected on classpath

In my case it also turned into a multi-hour debugging session. Trying to set up a more verbose logging turned out to be completely futile because the problem was that my application did not even start. Here's my context.xml:

<?xml version='1.0' encoding='utf-8'?>
<Context path="/rc2" docBase="rc2" antiResourceLocking="false" >
    <JarScanner>
        <JarScanFilter
            tldScan="spring-webmvc*.jar, spring-security-taglibs*.jar, jakarta.servlet.jsp.jstl*.jar"
            tldSkip="*.jar"
            <!-- my-own-app*.jar on the following line was missing! -->
            pluggabilityScan="${tomcat.util.scan.StandardJarScanFilter.jarsToScan}, my-own-app*.jar"
            pluggabilitySkip="*.jar"/>
    </JarScanner>
</Context>

The problem was that to speed up the application startup, I started skipping scanning of many JARs, unfortunately including my own application.

How to make lists contain only distinct element in Python?

The characteristics of sets in Python are that the data items in a set are unordered and duplicates are not allowed. If you try to add a data item to a set that already contains the data item, Python simply ignores it.

>>> l = ['a', 'a', 'bb', 'b', 'c', 'c', '10', '10', '8','8', 10, 10, 6, 10, 11.2, 11.2, 11, 11]
>>> distinct_l = set(l)
>>> print(distinct_l)
set(['a', '10', 'c', 'b', 6, 'bb', 10, 11, 11.2, '8'])

How to convert BigInteger to String in java

String input = "0101";
BigInteger x = new BigInteger ( input , 2 );
String output = x.toString(2);

Git clone particular version of remote repository

Probably git reset solves your problem.

git reset --hard -#commit hash-

Undefined index error PHP

this error occurred sometime method attribute ( valid passing method ) Error option : method="get" but called by $Fname = $_POST["name"]; or

       method="post" but  called by  $Fname = $_GET["name"];

More info visit http://www.doordie.co.in/index.php

Accessing Google Spreadsheets with C# using Google Data API

The most upvoted answer from @Kelly is no longer valid as @wescpy says. However after 2020-03-03 it will not work at all since the library used uses Google Sheets v3 API.

The Google Sheets v3 API will be shut down on March 3, 2020

https://developers.google.com/sheets/api/v3

This was announced 2019-09-10 by Google:

https://cloud.google.com/blog/products/g-suite/migrate-your-apps-use-latest-sheets-api

New code sample for Google Sheets v4 API:

Go to

https://developers.google.com/sheets/api/quickstart/dotnet

and generate credentials.json. Then install Google.Apis.Sheets.v4 NuGet and try the following sample:

Note that I got the error Unable to parse range: Class Data!A2:E with the example code but with my spreadsheet. Changing to Sheet1!A2:E worked however since my sheet was named that. Also worked with only A2:E.

using Google.Apis.Auth.OAuth2;
using Google.Apis.Sheets.v4;
using Google.Apis.Sheets.v4.Data;
using Google.Apis.Services;
using Google.Apis.Util.Store;
using System;
using System.Collections.Generic;
using System.IO;
using System.Threading;

namespace SheetsQuickstart
{
    class Program
    {
        // If modifying these scopes, delete your previously saved credentials
        // at ~/.credentials/sheets.googleapis.com-dotnet-quickstart.json
        static string[] Scopes = { SheetsService.Scope.SpreadsheetsReadonly };
        static string ApplicationName = "Google Sheets API .NET Quickstart";

        static void Main(string[] args)
        {
            UserCredential credential;

            using (var stream =
                new FileStream("credentials.json", FileMode.Open, FileAccess.Read))
            {
                // The file token.json stores the user's access and refresh tokens, and is created
                // automatically when the authorization flow completes for the first time.
                string credPath = "token.json";
                credential = GoogleWebAuthorizationBroker.AuthorizeAsync(
                    GoogleClientSecrets.Load(stream).Secrets,
                    Scopes,
                    "user",
                    CancellationToken.None,
                    new FileDataStore(credPath, true)).Result;
                Console.WriteLine("Credential file saved to: " + credPath);
            }

            // Create Google Sheets API service.
            var service = new SheetsService(new BaseClientService.Initializer()
            {
                HttpClientInitializer = credential,
                ApplicationName = ApplicationName,
            });

            // Define request parameters.
            String spreadsheetId = "1BxiMVs0XRA5nFMdKvBdBZjgmUUqptlbs74OgvE2upms";
            String range = "Class Data!A2:E";
            SpreadsheetsResource.ValuesResource.GetRequest request =
                    service.Spreadsheets.Values.Get(spreadsheetId, range);

            // Prints the names and majors of students in a sample spreadsheet:
            // https://docs.google.com/spreadsheets/d/1BxiMVs0XRA5nFMdKvBdBZjgmUUqptlbs74OgvE2upms/edit
            ValueRange response = request.Execute();
            IList<IList<Object>> values = response.Values;
            if (values != null && values.Count > 0)
            {
                Console.WriteLine("Name, Major");
                foreach (var row in values)
                {
                    // Print columns A and E, which correspond to indices 0 and 4.
                    Console.WriteLine("{0}, {1}", row[0], row[4]);
                }
            }
            else
            {
                Console.WriteLine("No data found.");
            }
            Console.Read();
        }
    }
}

Why binary_crossentropy and categorical_crossentropy give different performances for the same problem?

I came across an "inverted" issue — I was getting good results with categorical_crossentropy (with 2 classes) and poor with binary_crossentropy. It seems that problem was with wrong activation function. The correct settings were:

  • for binary_crossentropy: sigmoid activation, scalar target
  • for categorical_crossentropy: softmax activation, one-hot encoded target

Getting query parameters from react-router hash fragment

Simple js solution:

queryStringParse = function(string) {
    let parsed = {}
    if(string != '') {
        string = string.substring(string.indexOf('?')+1)
        let p1 = string.split('&')
        p1.map(function(value) {
            let params = value.split('=')
            parsed[params[0]] = params[1]
        });
    }
    return parsed
}

And you can call it from anywhere using:

var params = this.queryStringParse(this.props.location.search);

Hope this helps.

How to set Java SDK path in AndroidStudio?

Go to File>Project Structure>JDK location: Here, you have to set the directory path exactly same, in which you have installed the java version. Also, you have to mention the paths of SDK for project run on emulator successfully.

Why This Problem Occurs: It is due to the unsynchronized java version directory that should be available to Android Studio for java code compilance.

Set a button background image iPhone programmatically

In one line we can set image with this code

[buttonName setBackgroundImage:[UIImage imageNamed:@"imageName"] forState:UIControlStateNormal];

Parsing Json rest api response in C#

1> Add this namspace. using Newtonsoft.Json.Linq;

2> use this source code.

JObject joResponse = JObject.Parse(response);                   
JObject ojObject = (JObject)joResponse["response"];
JArray array= (JArray)ojObject ["chats"];
int id = Convert.ToInt32(array[0].toString());

What does int argc, char *argv[] mean?

The main function can have two parameters, argc and argv. argc is an integer (int) parameter, and it is the number of arguments passed to the program.

The program name is always the first argument, so there will be at least one argument to a program and the minimum value of argc will be one. But if a program has itself two arguments the value of argc will be three.

Parameter argv points to a string array and is called the argument vector. It is a one dimensional string array of function arguments.

Clone() vs Copy constructor- which is recommended in java

Clone is broken, so dont use it.

THE CLONE METHOD of the Object class is a somewhat magical method that does what no pure Java method could ever do: It produces an identical copy of its object. It has been present in the primordial Object superclass since the Beta-release days of the Java compiler*; and it, like all ancient magic, requires the appropriate incantation to prevent the spell from unexpectedly backfiring

Prefer a method that copies the object

Foo copyFoo (Foo foo){
  Foo f = new Foo();
  //for all properties in FOo
  f.set(foo.get());
  return f;
}

Read more http://adtmag.com/articles/2000/01/18/effective-javaeffective-cloning.aspx

Make xargs handle filenames that contain spaces

The xargs command takes white space characters (tabs, spaces, new lines) as delimiters.

You can narrow it down only for the new line characters ('\n') with -d option like this:

ls *.mp3 | xargs -d '\n' mplayer

It works only with GNU xargs.

For BSD systems, use the -0 option like this:

ls *.mp3 | xargs -0 mplayer

This method is simpler and works with the GNU xargs as well.

For MacOS:

ls *.mp3 | tr \\n \\0 | xargs -0 mplayer

What causes: "Notice: Uninitialized string offset" to appear?

The error may occur when the number of times you iterate the array is greater than the actual size of the array. for example:

 $one="909";
 for($i=0;$i<10;$i++)
    echo ' '.$one[$i];

will show the error. first case u can take the mod of i.. for example

function mod($i,$length){
  $m = $i % $size;
  if ($m > $size)
  mod($m,$size)
  return $m;
}

for($i=0;$i<10;$i++)
{
  $k=mod($i,3);
  echo ' '.$one[$k];
}

or might be it not an array (maybe it was a value and you tried to access it like an array) for example:

$k = 2;
$k[0];

Truncate/round whole number in JavaScript?

Travis Pessetto's answer along with mozey's trunc2 function were the only correct answers, considering how JavaScript represents very small or very large floating point numbers in scientific notation.

For example, parseInt(-2.2043642353916286e-15) will not correctly parse that input. Instead of returning 0 it will return -2.

This is the correct (and imho the least insane) way to do it:

function truncate(number)
{
    return number > 0
         ? Math.floor(number)
         : Math.ceil(number);
}

Bootstrap 4 - Responsive cards in card-columns

I realize this question was posted a while ago; nonetheless, Bootstrap v4.0 has card layout support out of the box. You can find the documentation here: Bootstrap Card Layouts.

I've gotten back into using Bootstrap for a recent project that relies heavily on the card layout UI. I've found success with the following implementation across the standard breakpoints:

_x000D_
_x000D_
<link href="https://unpkg.com/[email protected]/css/tachyons.min.css" rel="stylesheet"/>_x000D_
<link href="https://stackpath.bootstrapcdn.com/bootstrap/4.3.1/css/bootstrap.min.css" rel="stylesheet"/>_x000D_
<script src="https://cdnjs.cloudflare.com/ajax/libs/jquery/3.3.1/jquery.min.js"></script>_x000D_
<div class="flex justify-center" id="cars" v-cloak>_x000D_
    <!-- RELEVANT MARKUP BEGINS HERE -->_x000D_
    <div class="container mh0 w-100">_x000D_
        <div class="page-header text-center mb5">_x000D_
            <h1 class="avenir text-primary mb-0">Cars</h1>_x000D_
            <p class="text-secondary">Add and manage your cars for sale.</p>_x000D_
            <div class="header-button">_x000D_
                <button class="btn btn-outline-primary" @click="clickOpenAddCarModalButton">Add a car for sale</button>_x000D_
            </div>_x000D_
        </div>_x000D_
        <div class="container pa0 flex justify-center">_x000D_
            <div class="listings card-columns">_x000D_
                <div class="card mv2">_x000D_
                    <img src="https://farm4.staticflickr.com/3441/3361756632_8d84aa8560.jpg" class="card-img-top"_x000D_
                        alt="Mazda hatchback">_x000D_
                    <div class="card-body">_x000D_
                        <h5 class="card-title">Card title</h5>_x000D_
                        <p class="card-text">Some quick example text to build on the card title and make up the bulk of the card's_x000D_
                            content._x000D_
                        </p>_x000D_
                        <a href="#" class="btn btn-primary">Go somewhere</a>_x000D_
                    </div>_x000D_
                    <div class="card-footer">_x000D_
                        buttons here_x000D_
                    </div>_x000D_
                </div>_x000D_
                <div class="card mv2">_x000D_
                    <img src="https://farm4.staticflickr.com/3441/3361756632_8d84aa8560.jpg" class="card-img-top"_x000D_
                        alt="Mazda hatchback">_x000D_
                    <div class="card-body">_x000D_
                        <h5 class="card-title">Card title</h5>_x000D_
                        <p class="card-text">Some quick example text to build on the card title and make up the bulk of the card's_x000D_
                            content._x000D_
                        </p>_x000D_
                        <a href="#" class="btn btn-primary">Go somewhere</a>_x000D_
                    </div>_x000D_
                    <div class="card-footer">_x000D_
                        buttons here_x000D_
                    </div>_x000D_
                </div>_x000D_
                <div class="card mv2">_x000D_
                    <img src="https://farm4.staticflickr.com/3441/3361756632_8d84aa8560.jpg" class="card-img-top"_x000D_
                        alt="Mazda hatchback">_x000D_
                    <div class="card-body">_x000D_
                        <h5 class="card-title">Card title</h5>_x000D_
                        <p class="card-text">Some quick example text to build on the card title and make up the bulk of the card's_x000D_
                            content._x000D_
                        </p>_x000D_
                        <a href="#" class="btn btn-primary">Go somewhere</a>_x000D_
                    </div>_x000D_
                    <div class="card-footer">_x000D_
                        buttons here_x000D_
                    </div>_x000D_
                </div>_x000D_
                <div class="card mv2">_x000D_
                    <img src="https://farm4.staticflickr.com/3441/3361756632_8d84aa8560.jpg" class="card-img-top"_x000D_
                        alt="Mazda hatchback">_x000D_
                    <div class="card-body">_x000D_
                        <h5 class="card-title">Card title</h5>_x000D_
                        <p class="card-text">Some quick example text to build on the card title and make up the bulk of the card's_x000D_
                            content._x000D_
                        </p>_x000D_
                        <a href="#" class="btn btn-primary">Go somewhere</a>_x000D_
                    </div>_x000D_
                    <div class="card-footer">_x000D_
                        buttons here_x000D_
                    </div>_x000D_
                </div>_x000D_
                <div class="card mv2">_x000D_
                    <img src="https://farm4.staticflickr.com/3441/3361756632_8d84aa8560.jpg" class="card-img-top"_x000D_
                        alt="Mazda hatchback">_x000D_
                    <div class="card-body">_x000D_
                        <h5 class="card-title">Card title</h5>_x000D_
                        <p class="card-text">Some quick example text to build on the card title and make up the bulk of the card's_x000D_
                            content._x000D_
                        </p>_x000D_
                        <a href="#" class="btn btn-primary">Go somewhere</a>_x000D_
                    </div>_x000D_
                    <div class="card-footer">_x000D_
                        buttons here_x000D_
                    </div>_x000D_
                </div>_x000D_
            </div>_x000D_
        </div>_x000D_
    </div>_x000D_
</div>
_x000D_
_x000D_
_x000D_

After trying both the Bootstrap .card-group and .card-deck card layout classes with quirky results at best across the standard breakpoints, I finally decided to give the .card-columns class a shot. And it worked!

Your results may vary, but .card-columns seems to be the most stable implementation here.

"&" meaning after variable type

It means you're passing the variable by reference.

In fact, in a declaration of a type, it means reference, just like:

int x = 42;
int& y = x;

declares a reference to x, called y.

Where can I find the .apk file on my device, when I download any app and install?

You can do that I believe. It needs root permission. If you want to know where your apk files are stored, open a emulator and then go to

DDMS>File Explorer-> you can see a directory by name "data" -> Click on it and you will see a "app" folder.

Your apks are stored there. In fact just copying a apk directly to the folder works for me with emulators.

How can I start InternetExplorerDriver using Selenium WebDriver

  1. You must set Protected Mode settings for each zone to be the same value.
  2. Enhanced Protected Mode for all zones must be same. (I prefer it to be disabled as this is the requirement for IE 10 and higher.)

Additionally, "Enhanced Protected Mode" must be disabled for IE 10 and higher. This option is found in the Advanced tab of the Internet Options dialog.

How to do above steps???

Have look at this video: http://screencast.com/t/5nlxsrje4I . I have showed the steps.

Source: https://code.google.com/p/selenium/wiki/InternetExplorerDriver#Required_Configuration

Hope this helps. Thank you :)

How to set HTTP headers (for cache-control)?

For Apache server, you should check mod_expires for setting Expires and Cache-Control headers.

Alternatively, you can use Header directive to add Cache-Control on your own:

Header set Cache-Control "max-age=290304000, public"

Round to at most 2 decimal places (only if necessary)

MarkG's answer is the correct one. Here's a generic extension for any number of decimal places.

Number.prototype.round = function(places) {
  return +(Math.round(this + "e+" + places)  + "e-" + places);
}

Usage:

var n = 1.7777;    
n.round(2); // 1.78

Unit test:

it.only('should round floats to 2 places', function() {

  var cases = [
    { n: 10,      e: 10,    p:2 },
    { n: 1.7777,  e: 1.78,  p:2 },
    { n: 1.005,   e: 1.01,  p:2 },
    { n: 1.005,   e: 1,     p:0 },
    { n: 1.77777, e: 1.8,   p:1 }
  ]

  cases.forEach(function(testCase) {
    var r = testCase.n.round(testCase.p);
    assert.equal(r, testCase.e, 'didn\'t get right number');
  });
})

How long will my session last?

This is the one. The session will last for 1440 seconds (24 minutes).

session.gc_maxlifetime  1440    1440

How to see the actual Oracle SQL statement that is being executed

On the data dictionary side there are a lot of tools you can use to such as Schema Spy

To look at what queries are running look at views sys.v_$sql and sys.v_$sqltext. You will also need access to sys.all_users

One thing to note that queries that use parameters will show up once with entries like

and TABLETYPE=’:b16’

while others that dont will show up multiple times such as:

and TABLETYPE=’MT’

An example of these tables in action is the following SQL to find the top 20 diskread hogs. You could change this by removing the WHERE rownum <= 20 and maybe add ORDER BY module. You often find the module will give you a bog clue as to what software is running the query (eg: "TOAD 9.0.1.8", "JDBC Thin Client", "runcbl@somebox (TNS V1-V3)" etc)

SELECT 
 module, 
 sql_text, 
 username, 
 disk_reads_per_exec, 
 buffer_gets, 
 disk_reads, 
 parse_calls, 
 sorts, 
 executions, 
 rows_processed, 
 hit_ratio, 
 first_load_time, 
 sharable_mem, 
 persistent_mem, 
 runtime_mem, 
 cpu_time, 
 elapsed_time, 
 address, 
 hash_value 
FROM 
  (SELECT
   module, 
   sql_text , 
   u.username , 
   round((s.disk_reads/decode(s.executions,0,1, s.executions)),2)  disk_reads_per_exec, 
   s.disk_reads , 
   s.buffer_gets , 
   s.parse_calls , 
   s.sorts , 
   s.executions , 
   s.rows_processed , 
   100 - round(100 *  s.disk_reads/greatest(s.buffer_gets,1),2) hit_ratio, 
   s.first_load_time , 
   sharable_mem , 
   persistent_mem , 
   runtime_mem, 
   cpu_time, 
   elapsed_time, 
   address, 
   hash_value 
  FROM
   sys.v_$sql s, 
   sys.all_users u 
  WHERE
   s.parsing_user_id=u.user_id 
   and UPPER(u.username) not in ('SYS','SYSTEM') 
  ORDER BY
   4 desc) 
WHERE
 rownum <= 20;

Note that if the query is long .. you will have to query v_$sqltext. This stores the whole query. You will have to look up the ADDRESS and HASH_VALUE and pick up all the pieces. Eg:

SELECT
 *
FROM
 sys.v_$sqltext
WHERE
 address = 'C0000000372B3C28'
 and hash_value = '1272580459'
ORDER BY 
 address, hash_value, command_type, piece
;

PowerShell: Format-Table without headers

The -HideTableHeaders parameter unfortunately still causes the empty lines to be printed (and table headers appearently are still considered for column width). The only way I know that could reliably work here would be to format the output yourself:

| % { '{0,10} {1,20} {2,20}' -f $_.Operation,$_.AttributeName,$_.AttributeValue }

How to check if a Docker image with a specific tag exist locally?

In bash script I do this to check if image exists by tag :

IMAGE_NAME="mysql:5.6"

if docker image ls -a "$IMAGE_NAME" | grep -Fq "$IMAGE_NAME" 1>/dev/null; then
echo "could found image $IMAGE_NAME..."
fi

Example script above checks if mysql image with 5.6 tag exists. If you want just check if any mysql image exists without specific version then just pass repository name without tag as this :

IMAGE_NAME="mysql"

C++: Where to initialize variables in constructor

Option 1 allows you to use a place specified exactly for explicitly initializing member variables.

Algorithm: efficient way to remove duplicate integers from an array

You could do this in a single traversal, if you are willing to sacrifice memory. You can simply tally whether you have seen an integer or not in a hash/associative array. If you have already seen a number, remove it as you go, or better yet, move numbers you have not seen into a new array, avoiding any shifting in the original array.

In Perl:

foreach $i (@myary) {
    if(!defined $seen{$i}) {
        $seen{$i} = 1;
        push @newary, $i;
    }
}

How can I send mail from an iPhone application

Swift 2.0

func mailComposeController(controller: MFMailComposeViewController, didFinishWithResult result: MFMailComposeResult, error: NSError?){
    if let error = error{
        print("Error: \(error)")
    }else{
        //NO Error
        //------------------------------------------------
        var feedbackMsg = ""

        switch result.rawValue {
        case MFMailComposeResultCancelled.rawValue:
            feedbackMsg = "Mail Cancelled"
        case MFMailComposeResultSaved.rawValue:
            feedbackMsg = "Mail Saved"
        case MFMailComposeResultSent.rawValue:
            feedbackMsg = "Mail Sent"
        case MFMailComposeResultFailed.rawValue:
            feedbackMsg = "Mail Failed"
        default:
            feedbackMsg = ""
        }

        print("Mail: \(feedbackMsg)")

        //------------------------------------------------
    }
}

.Net picking wrong referenced assembly version

This error was somewhat misleading - I was loading some DLLs that required x64 architecture to be specified. In the .csproj file:

<PropertyGroup Condition="'$(Configuration)|$(Platform)' == 'Release-ABC|AnyCPU'">
    <OutputPath>bin\Release-ABC</OutputPath>
    <PlatformTarget>x64</PlatformTarget>
</PropertyGroup>

A missing PlatformTarget caused this error.

Django Admin - change header 'Django administration' text

you do not need to change any template for this work you just need to update the settings.py of your project. Go to the bottom of the settings.py and define this.

admin.site.site_header = 'My Site Admin'

In this way you would be able to change the header of the of the Django admin. Moreover you can read more about Django Admin customization and settings on the following link.

Django Admin Documentation

How to implement DrawerArrowToggle from Android appcompat v7 21 library

If you are using the Support Library provided DrawerLayout as suggested in the Creating a navigation drawer training, you can use the newly added android.support.v7.app.ActionBarDrawerToggle (note: different from the now deprecated android.support.v4.app.ActionBarDrawerToggle):

shows a Hamburger icon when drawer is closed and an arrow when drawer is open. It animates between these two states as the drawer opens.

While the training hasn't been updated to take the deprecation/new class into account, you should be able to use it almost exactly the same code - the only difference in implementing it is the constructor.

Bootstrap get div to align in the center

When I align elements in center I use the bootstrap class text-center:

<div class="text-center">Centered content goes here</div>

Extract file name from path, no matter what the os/path format

In your example you will also need to strip slash from right the right side to return c:

>>> import os
>>> path = 'a/b/c/'
>>> path = path.rstrip(os.sep) # strip the slash from the right side
>>> os.path.basename(path)
'c'

Second level:

>>> os.path.filename(os.path.dirname(path))
'b'

update: I think lazyr has provided the right answer. My code will not work with windows-like paths on unix systems and vice versus with unix-like paths on windows system.

What is the idiomatic Go equivalent of C's ternary operator?

As pointed out (and hopefully unsurprisingly), using if+else is indeed the idiomatic way to do conditionals in Go.

In addition to the full blown var+if+else block of code, though, this spelling is also used often:

index := val
if val <= 0 {
    index = -val
}

and if you have a block of code that is repetitive enough, such as the equivalent of int value = a <= b ? a : b, you can create a function to hold it:

func min(a, b int) int {
    if a <= b {
        return a
    }
    return b
}

...

value := min(a, b)

The compiler will inline such simple functions, so it's fast, more clear, and shorter.

Using an index to get an item, Python

Same as any other language, just pass index number of element that you want to retrieve.

#!/usr/bin/env python
x = [2,3,4,5,6,7]
print(x[5])

UICollectionView Set number of columns

the perfect solution is to Using UICollectionViewDelegateFlowLayout but you can easily so calculate the cell width and divided on the wanted number of columns you want

the tricky is to make the width with no fraction

(UICollectionViewLayout*)collectionViewLayout sizeForItemAtIndexPath:(NSIndexPath *)indexPath

{
 CGFloat screenWidth = self.view.frame.size.width;
   CGFloat marginWidth = (screenWidth - collectionView.frame.size.width);


   CGFloat cellWith = (collectionView.frame.size.width - marginWidth )/3;
   cellWith= floorf(cellWith);




  CGSize retval = CGSizeMake(cellWith,cellWith);


  return retval;}

What is the default initialization of an array in Java?

From the Java Language Specification:

  • Each class variable, instance variable, or array component is initialized with a default value when it is created (§15.9, §15.10):

    • For type byte, the default value is zero, that is, the value of (byte)0.
    • For type short, the default value is zero, that is, the value of (short)0.
    • For type int, the default value is zero, that is, 0.
    • For type long, the default value is zero, that is, 0L.
    • For type float, the default value is positive zero, that is, 0.0f.
    • For type double, the default value is positive zero, that is, 0.0d.
    • For type char, the default value is the null character, that is, '\u0000'.
    • For type boolean, the default value is false.
    • For all reference types (§4.3), the default value is null.

Resource u'tokenizers/punkt/english.pickle' not found

I got the solution:

import nltk
nltk.download()

once the NLTK Downloader starts

d) Download l) List u) Update c) Config h) Help q) Quit

Downloader> d

Download which package (l=list; x=cancel)? Identifier> punkt

Must issue a STARTTLS command first

Google now has a feature stating that it won't allow insecure devices to send emails. When I ran my program it came up with the error in the first post. I had to go into my account and allow insecure apps to send emails, which I did by clicking on my account, going into the security tab, and allowing insecure apps to use my gmail.

Iterating over dictionaries using 'for' loops

This is a very common looping idiom. in is an operator. For when to use for key in dict and when it must be for key in dict.keys() see David Goodger's Idiomatic Python article (archived copy).

pip3: command not found but python3-pip is already installed

You can make symbolic link to you pip3:

sudo ln -s $(which pip3) /usr/bin/pip3

It helps me in RHEL 7.6

Increasing the Command Timeout for SQL command

Since it takes 2 mins to respond, you can increase the timeout to 3 mins by adding the below code

scGetruntotals.CommandTimeout = 180;

Note : the parameter value is in seconds.

Why dividing two integers doesn't get a float?

Probably the best reason is because 0xfffffffffffffff/15 would give you a horribly wrong answer...

How to escape braces (curly brackets) in a format string in .NET

Yes to output { in string.Format you have to escape it like this {{

So this

String val = "1,2,3";
String.Format(" foo {{{0}}}", val);

will output "foo {1,2,3}".

BUT you have to know about a design bug in C# which is that by going on the above logic you would assume this below code will print {24.00}

int i = 24;
string str = String.Format("{{{0:N}}}", i); //gives '{N}' instead of {24.00}

But this prints {N}. This is because the way C# parses escape sequences and format characters. To get the desired value in the above case you have to use this instead.

String.Format("{0}{1:N}{2}", "{", i, "}") //evaluates to {24.00}

Reference Articles String.Format gottach and String Formatting FAQ

How do you change the text in the Titlebar in Windows Forms?

using System;
using System.Collections.Generic;
using System.ComponentModel;
using System.Data;
using System.Drawing;
using System.Linq;
using System.Text;
using System.Windows.Forms;

namespace WindowsFormsApplication1
{
    public partial class Form1 : Form
    {
        public Form1()
        {
            InitializeComponent();
        }

        private void Form1_Load(object sender, EventArgs e)
        {

        }

        //private void Form1_Load(object sender, EventArgs e)
        //{

        //    // Instantiate a new instance of Form1.
        //    Form1 f1 = new Form1();
        //    f1.Text = "zzzzzzz";

        //}
    }

    class MainApplication
    {
        public static void Main()
        {
            // Instantiate a new instance of Form1.
            Form1 f1 = new Form1();
            // Display a messagebox. This shows the application 
            // is running, yet there is nothing shown to the user. 
            // This is the point at which you customize your form.
            System.Windows.Forms.MessageBox.Show("The application "
               + "is running now, but no forms have been shown.");
            // Customize the form.
            f1.Text = "Running Form";
            // Show the instance of the form modally.
            f1.ShowDialog();
        }
    }

}

Do you know the Maven profile for mvnrepository.com?

Place this in the ~/.m2/settings.xml or custom file to be run with $ mvn -s custom-settings.xml install

<settings xmlns="http://maven.apache.org/SETTINGS/1.0.0"
xmlns:xsi="http://www.w3.org/2001/XMLSchema-instance"
xsi:schemaLocation="http://maven.apache.org/SETTINGS/1.0.0
https://maven.apache.org/xsd/settings-1.0.0.xsd">
<localRepository>${user.home}/.m2/repository</localRepository>
<interactiveMode/>
<offline/>
<pluginGroups/>
<profiles>
    <profile>
        <repositories>
            <repository>
                <id>mvnrepository</id>
                <name>mvnrepository</name>
                <url>http://www.mvnrepository.com</url>
            </repository>
        </repositories>
    </profile>
</profiles>
<activeProfiles>
    <activeProfile>mvnrepository</activeProfile>
</activeProfiles>
</settings>

AngularJS: Can't I set a variable value on ng-click?

If you are using latest versions of Angular (2/5/6) :

In your component.ts

//x.component.ts
prefs = false;

hidePrefs(){
   this.prefs = true;
}

ubuntu "No space left on device" but there is tons of space

It's possible that you've run out of memory or some space elsewhere and it prompted the system to mount an overflow filesystem, and for whatever reason, it's not going away.

Try unmounting the overflow partition:

umount /tmp

or

umount overflow

jquery: change the URL address without redirecting?

NOTE: history.pushState() is now supported - see other answers.

You cannot change the whole url without redirecting, what you can do instead is change the hash.

The hash is the part of the url that goes after the # symbol. That was initially intended to direct you (locally) to sections of your HTML document, but you can read and modify it through javascript to use it somewhat like a global variable.


If applied well, this technique is useful in two ways:

  1. the browser history will remember each different step you took (since the url+hash changed)
  2. you can have an address which links not only to a particular html document, but also gives your javascript a clue about what to do. That means you end up pointing to a state inside your web app.

To change the hash you can do:

document.location.hash = "show_picture";

To watch for hash changes you have to do something like:

window.onhashchange = function(){
    var what_to_do = document.location.hash;    
    if (what_to_do=="#show_picture")
        show_picture();
}

Of course the hash is just a string, so you can do pretty much what you like with it. For example you can put a whole object there if you use JSON to stringify it.

There are very good JQuery libraries to do advanced things with that.

Uploading images using Node.js, Express, and Mongoose

Since you're using express, just add bodyParser:

app.use(express.bodyParser());

then your route automatically has access to the uploaded file(s) in req.files:

app.post('/todo/create', function (req, res) {
    // TODO: move and rename the file using req.files.path & .name)
    res.send(console.dir(req.files));  // DEBUG: display available fields
});

If you name the input control "todo" like this (in Jade):

form(action="/todo/create", method="POST", enctype="multipart/form-data")
    input(type='file', name='todo')
    button(type='submit') New

Then the uploaded file is ready by the time you get the path and original filename in 'files.todo':

  • req.files.todo.path, and
  • req.files.todo.name

other useful req.files properties:

  • size (in bytes)
  • type (e.g., 'image/png')
  • lastModifiedate
  • _writeStream.encoding (e.g, 'binary')

Correct file permissions for WordPress

Define in wp_config file.

/var/www/html/Your-Project-File/wp-config.php

define( 'FS_METHOD', 'direct' );

chown - changes ownership of files/dirs. Ie. owner of the file/dir changes to the specified one, but it doesn't modify permissions.

sudo chown -R www-data:www-data /var/www

iPhone is not available. Please reconnect the device

Going to Window ? Devices and Simulators will give you a better idea of what's going on. In my case, I had to update the iPhone since Xcode updated overnight and stopped supporting my iPhone.

Enter image description here

How to create an Explorer-like folder browser control?

Microsoft provides a walkthrough for creating a Windows Explorer style interface in C#.

There are also several examples on Code Project and other sites. Immediate examples are Explorer Tree, My Explorer, File Browser and Advanced File Explorer but there are others. Explorer Tree seems to look the best from the brief glance I took.

I used the search term windows explorer tree view C# in Google to find these links.

Postgres: How to do Composite keys?

Your compound PRIMARY KEY specification already does what you want. Omit the line that's giving you a syntax error, and omit the redundant CONSTRAINT (already implied), too:

 CREATE TABLE tags
      (
               question_id INTEGER NOT NULL,
               tag_id SERIAL NOT NULL,
               tag1 VARCHAR(20),
               tag2 VARCHAR(20),
               tag3 VARCHAR(20),
               PRIMARY KEY(question_id, tag_id)
      );

NOTICE:  CREATE TABLE will create implicit sequence "tags_tag_id_seq" for serial column "tags.tag_id"
    NOTICE:  CREATE TABLE / PRIMARY KEY will create implicit index "tags_pkey" for table "tags"
    CREATE TABLE
    pg=> \d tags
                                         Table "public.tags"
       Column    |         Type          |                       Modifiers       
    -------------+-----------------------+-------------------------------------------------------
     question_id | integer               | not null
     tag_id      | integer               | not null default nextval('tags_tag_id_seq'::regclass)
     tag1        | character varying(20) |
     tag2        | character varying(20) |
     tag3        | character varying(20) |
    Indexes:
        "tags_pkey" PRIMARY KEY, btree (question_id, tag_id)

Apply CSS Style to child elements

Here is some code that I recently wrote. I think that it provides a basic explanation of combining class/ID names with pseudoclasses.

_x000D_
_x000D_
.content {_x000D_
  width: 800px;_x000D_
  border: 1px solid black;_x000D_
  border-radius: 10px;_x000D_
  box-shadow: 0 0 5px 2px grey;_x000D_
  margin: 30px auto 20px auto;_x000D_
  /*height:200px;*/_x000D_
_x000D_
}_x000D_
p.red {_x000D_
  color: red;_x000D_
}_x000D_
p.blue {_x000D_
  color: blue;_x000D_
}_x000D_
p#orange {_x000D_
  color: orange;_x000D_
}_x000D_
p#green {_x000D_
  color: green;_x000D_
}
_x000D_
<!DOCTYPE html>_x000D_
<html>_x000D_
_x000D_
<head>_x000D_
  <title>Class practice</title>_x000D_
  <link href="wrench_favicon.ico" rel="icon" type="image/x-icon" />_x000D_
</head>_x000D_
_x000D_
<body>_x000D_
  <div class="content">_x000D_
    <p id="orange">orange</p>_x000D_
    <p id="green">green</p>_x000D_
    <p class="red">red</p>_x000D_
    <p class="blue">blue</p>_x000D_
  </div>_x000D_
</body>_x000D_
_x000D_
</html>
_x000D_
_x000D_
_x000D_

Return in Scala

This topic is actually a little more complicated as described in the answers so far. This blogpost by Rob Norris explains it in more detail and gives examples on when using return will actually break your code (or at least have non-obvious effects).

At this point let me just quote the essence of the post. The most important statement is right in the beginning. Print this as a poster and put it to your wall :-)

The return keyword is not “optional” or “inferred”; it changes the meaning of your program, and you should never use it.

It gives one example, where it actually breaks something, when you inline a function

// Inline add and addR
def sum(ns: Int*): Int = ns.foldLeft(0)((n, m) => n + m) // inlined add

scala> sum(33, 42, 99)
res2: Int = 174 // alright

def sumR(ns: Int*): Int = ns.foldLeft(0)((n, m) => return n + m) // inlined addR

scala> sumR(33, 42, 99)
res3: Int = 33 // um.

because

A return expression, when evaluated, abandons the current computation and returns to the caller of the method in which return appears.

This is only one of the examples given in the linked post and it's the easiest to understand. There're more and I highly encourage you, to go there, read and understand.

When you come from imperative languages like Java, this might seem odd at first, but once you get used to this style it will make sense. Let me close with another quote:

If you find yourself in a situation where you think you want to return early, you need to re-think the way you have defined your computation.

How to run python script with elevated privilege on windows

I wanted a more enhanced version so I ended up with a module which allows: UAC request if needed, printing and logging from nonprivileged instance (uses ipc and a network port) and some other candies. usage is just insert elevateme() in your script: in nonprivileged it listen for privileged print/logs and then exits returning false, in privileged instance it returns true immediately. Supports pyinstaller.

prototype:

# xlogger : a logger in the server/nonprivileged script
# tport : open port of communication, 0 for no comm [printf in nonprivileged window or silent]
# redir : redirect stdout and stderr from privileged instance
#errFile : redirect stderr to file from privileged instance
def elevateme(xlogger=None, tport=6000, redir=True, errFile=False):

winadmin.py

#!/usr/bin/env python
# -*- coding: utf-8; mode: python; py-indent-offset: 4; indent-tabs-mode: nil -*-
# vim: fileencoding=utf-8 tabstop=4 expandtab shiftwidth=4

# (C) COPYRIGHT © Preston Landers 2010
# (C) COPYRIGHT © Matteo Azzali 2020
# Released under the same license as Python 2.6.5/3.7


import sys, os
from traceback import print_exc
from multiprocessing.connection import Listener, Client
import win32event #win32com.shell.shell, win32process
import builtins as __builtin__ # python3

# debug suffixes for remote printing
dbz=["","","",""] #["J:","K:", "G:", "D:"]
LOGTAG="LOGME:"

wrconn = None


#fake logger for message sending
class fakelogger:
    def __init__(self, xlogger=None):
        self.lg = xlogger
    def write(self, a):
        global wrconn
        if wrconn is not None:
            wrconn.send(LOGTAG+a)
        elif self.lg is not None:
            self.lg.write(a)
        else:
            print(LOGTAG+a)
        

class Writer():
    wzconn=None
    counter = 0
    def __init__(self, tport=6000,authkey=b'secret password'):
        global wrconn
        if wrconn is None:
            address = ('localhost', tport)
            try:
                wrconn = Client(address, authkey=authkey)
            except:
                wrconn = None
            wzconn = wrconn
            self.wrconn = wrconn
        self.__class__.counter+=1
        
    def __del__(self):
        self.__class__.counter-=1
        if self.__class__.counter == 0 and wrconn is not None:
            import time
            time.sleep(0.1) # slows deletion but is enough to print stderr
            wrconn.send('close')
            wrconn.close()
    
    def sendx(cls, mesg):
        cls.wzconn.send(msg)
        
    def sendw(self, mesg):
        self.wrconn.send(msg)
        

#fake file to be passed as stdout and stderr
class connFile():
    def __init__(self, thekind="out", tport=6000):
        self.cnt = 0
        self.old=""
        self.vg=Writer(tport)
        if thekind == "out":
            self.kind=sys.__stdout__
        else:
            self.kind=sys.__stderr__
        
    def write(self, *args, **kwargs):
        global wrconn
        global dbz
        from io import StringIO # # Python2 use: from cStringIO import StringIO
        mystdout = StringIO()
        self.cnt+=1
        __builtin__.print(*args, **kwargs, file=mystdout, end = '')
        
        #handles "\n" wherever it is, however usually is or string or \n
        if "\n" not in mystdout.getvalue():
            if mystdout.getvalue() != "\n":
                #__builtin__.print("A:",mystdout.getvalue(), file=self.kind, end='')
                self.old += mystdout.getvalue()
            else:
                #__builtin__.print("B:",mystdout.getvalue(), file=self.kind, end='')
                if wrconn is not None:
                    wrconn.send(dbz[1]+self.old)
                else:
                    __builtin__.print(dbz[2]+self.old+ mystdout.getvalue(), file=self.kind, end='')
                    self.kind.flush()
                self.old=""
        else:
                vv = mystdout.getvalue().split("\n")
                #__builtin__.print("V:",vv, file=self.kind, end='')
                for el in vv[:-1]:
                    if wrconn is not None:
                        wrconn.send(dbz[0]+self.old+el)
                        self.old = ""
                    else:
                        __builtin__.print(dbz[3]+self.old+ el+"\n", file=self.kind, end='')
                        self.kind.flush()
                        self.old=""
                self.old=vv[-1]

    def open(self):
        pass
    def close(self):
        pass
    def flush(self):
        pass
        
        
def isUserAdmin():
    if os.name == 'nt':
        import ctypes
        # WARNING: requires Windows XP SP2 or higher!
        try:
            return ctypes.windll.shell32.IsUserAnAdmin()
        except:
            traceback.print_exc()
            print ("Admin check failed, assuming not an admin.")
            return False
    elif os.name == 'posix':
        # Check for root on Posix
        return os.getuid() == 0
    else:
        print("Unsupported operating system for this module: %s" % (os.name,))
        exit()
        #raise (RuntimeError, "Unsupported operating system for this module: %s" % (os.name,))

def runAsAdmin(cmdLine=None, wait=True, hidden=False):

    if os.name != 'nt':
        raise (RuntimeError, "This function is only implemented on Windows.")

    import win32api, win32con, win32process
    from win32com.shell.shell import ShellExecuteEx

    python_exe = sys.executable
    arb=""
    if cmdLine is None:
        cmdLine = [python_exe] + sys.argv
    elif not isinstance(cmdLine, (tuple, list)):
        if isinstance(cmdLine, (str)):
            arb=cmdLine
            cmdLine = [python_exe] + sys.argv
            print("original user", arb)
        else:
            raise( ValueError, "cmdLine is not a sequence.")
    cmd = '"%s"' % (cmdLine[0],)

    params = " ".join(['"%s"' % (x,) for x in cmdLine[1:]])
    if len(arb) > 0:
        params += " "+arb
    cmdDir = ''
    if hidden:
        showCmd = win32con.SW_HIDE
    else:
        showCmd = win32con.SW_SHOWNORMAL
    lpVerb = 'runas'  # causes UAC elevation prompt.

    # print "Running", cmd, params

    # ShellExecute() doesn't seem to allow us to fetch the PID or handle
    # of the process, so we can't get anything useful from it. Therefore
    # the more complex ShellExecuteEx() must be used.

    # procHandle = win32api.ShellExecute(0, lpVerb, cmd, params, cmdDir, showCmd)

    procInfo = ShellExecuteEx(nShow=showCmd,
                              fMask=64,
                              lpVerb=lpVerb,
                              lpFile=cmd,
                              lpParameters=params)

    if wait:
        procHandle = procInfo['hProcess']    
        obj = win32event.WaitForSingleObject(procHandle, win32event.INFINITE)
        rc = win32process.GetExitCodeProcess(procHandle)
        #print "Process handle %s returned code %s" % (procHandle, rc)
    else:
        rc = procInfo['hProcess']

    return rc


# xlogger : a logger in the server/nonprivileged script
# tport : open port of communication, 0 for no comm [printf in nonprivileged window or silent]
# redir : redirect stdout and stderr from privileged instance
#errFile : redirect stderr to file from privileged instance
def elevateme(xlogger=None, tport=6000, redir=True, errFile=False):
    global dbz
    if not isUserAdmin():
        print ("You're not an admin.", os.getpid(), "params: ", sys.argv)

        import getpass
        uname = getpass.getuser()
        
        if (tport> 0):
            address = ('localhost', tport)     # family is deduced to be 'AF_INET'
            listener = Listener(address, authkey=b'secret password')
        rc = runAsAdmin(uname, wait=False, hidden=True)
        if (tport> 0):
            hr = win32event.WaitForSingleObject(rc, 40)
            conn = listener.accept()
            print ('connection accepted from', listener.last_accepted)
            sys.stdout.flush()
            while True:
                msg = conn.recv()
                # do something with msg
                if msg == 'close':
                    conn.close()
                    break
                else:
                    if msg.startswith(dbz[0]+LOGTAG):
                        if xlogger != None:
                            xlogger.write(msg[len(LOGTAG):])
                        else:
                            print("Missing a logger")
                    else:
                        print(msg)
                    sys.stdout.flush()
            listener.close()
        else: #no port connection, its silent
            WaitForSingleObject(rc, INFINITE);
        return False
    else:
        #redirect prints stdout on  master, errors in error.txt
        print("HIADM")
        sys.stdout.flush()
        if (tport > 0) and (redir):
            vox= connFile(tport=tport)
            sys.stdout=vox
            if not errFile:
                sys.stderr=vox
            else:
                vfrs=open("errFile.txt","w")
                sys.stderr=vfrs
            
            #print("HI ADMIN")
        return True


def test():
    rc = 0
    if not isUserAdmin():
        print ("You're not an admin.", os.getpid(), "params: ", sys.argv)
        sys.stdout.flush()
        #rc = runAsAdmin(["c:\\Windows\\notepad.exe"])
        rc = runAsAdmin()
    else:
        print ("You are an admin!", os.getpid(), "params: ", sys.argv)
        rc = 0
    x = raw_input('Press Enter to exit.')
    return rc
    
if __name__ == "__main__":
    sys.exit(test())

What is meant by immutable?

Immutable Objects

An object is considered immutable if its state cannot change after it is constructed. Maximum reliance on immutable objects is widely accepted as a sound strategy for creating simple, reliable code.

Immutable objects are particularly useful in concurrent applications. Since they cannot change state, they cannot be corrupted by thread interference or observed in an inconsistent state.

Programmers are often reluctant to employ immutable objects, because they worry about the cost of creating a new object as opposed to updating an object in place. The impact of object creation is often overestimated, and can be offset by some of the efficiencies associated with immutable objects. These include decreased overhead due to garbage collection, and the elimination of code needed to protect mutable objects from corruption.

The following subsections take a class whose instances are mutable and derives a class with immutable instances from it. In so doing, they give general rules for this kind of conversion and demonstrate some of the advantages of immutable objects.

Source

Accessing Arrays inside Arrays In PHP

If $a is the array that's passed, $a[76][0]['id'] should give '76' and $a[76][1]['id'] should give '81', but I can't test as I don't have PHP installed on this machine.

Character Limit on Instagram Usernames

Limit - 30 symbols. Username must contains only letters, numbers, periods and underscores.

How to tell if string starts with a number with Python?

Python's string library has isdigit() method:

string[0].isdigit()

CSS selector for text input fields?

input[type=text]

This will select all the input type text in a web-page.

Java best way for string find and replace?

One possibility, reducing the longer form before expanding all:

string.replaceAll("Milan Vasic", "Milan").replaceAll("Milan", "Milan Vasic")

Another way, treating Vasic as optional:

string.replaceAll("Milan( Vasic)?", "Milan Vasic")

Others have described solutions based on lookahead or alternation.

How can I change Mac OS's default Java VM returned from /usr/libexec/java_home

I think JAVA_HOME is the best you can do. The command-line tools like java and javac will respect that environment variable, you can use /usr/libexec/java_home -v '1.7*' to give you a suitable value to put into JAVA_HOME in order to make command line tools use Java 7.

export JAVA_HOME="`/usr/libexec/java_home -v '1.7*'`"

But standard double-clickable application bundles don't use JDKs installed under /Library/Java at all. Old-style .app bundles using Apple's JavaApplicationStub will use Apple Java 6 from /System/Library/Frameworks, and new-style ones built with AppBundler without a bundled JRE will use the "public" JRE in /Library/Internet Plug-Ins/JavaAppletPlugin.plugin/Contents/Home - that's hard-coded in the stub code and can't be changed, and you can't have two different public JREs installed at the same time.


Edit: I've had a look at VisualVM specifically, assuming you're using the "application bundle" version from the download page, and this particular app is not an AppBundler application, instead its main executable is a shell script that calls a number of other shell scripts and reads various configuration files. It defaults to picking the newest JDK from /Library/Java as long as that is 7u10 or later, or uses Java 6 if your Java 7 installation is update 9 or earlier. But unravelling the logic in the shell scripts it looks to me like you can specify a particular JDK using a configuration file.

Create a text file ~/Library/Application Support/VisualVM/1.3.6/etc/visualvm.conf (replace 1.3.6 with whatever version of VisualVM you're using) containing the line

visualvm_jdkhome="`/usr/libexec/java_home -v '1.7*'`"

and this will force it to choose Java 7 instead of 8.

Python Git Module experiences?

PTBNL's Answer is quite perfect for me. I make a little more for Windows user.

import time
import subprocess
def gitAdd(fileName, repoDir):
    cmd = 'git add ' + fileName
    pipe = subprocess.Popen(cmd, shell=True, cwd=repoDir,stdout = subprocess.PIPE,stderr = subprocess.PIPE )
    (out, error) = pipe.communicate()
    print out,error
    pipe.wait()
    return 

def gitCommit(commitMessage, repoDir):
    cmd = 'git commit -am "%s"'%commitMessage
    pipe = subprocess.Popen(cmd, shell=True, cwd=repoDir,stdout = subprocess.PIPE,stderr = subprocess.PIPE )
    (out, error) = pipe.communicate()
    print out,error
    pipe.wait()
    return 
def gitPush(repoDir):
    cmd = 'git push '
    pipe = subprocess.Popen(cmd, shell=True, cwd=repoDir,stdout = subprocess.PIPE,stderr = subprocess.PIPE )
    (out, error) = pipe.communicate()
    pipe.wait()
    return 

temp=time.localtime(time.time())
uploaddate= str(temp[0])+'_'+str(temp[1])+'_'+str(temp[2])+'_'+str(temp[3])+'_'+str(temp[4])

repoDir='d:\\c_Billy\\vfat\\Programming\\Projector\\billyccm' # your git repository , windows your need to use double backslash for right directory.
gitAdd('.',repoDir )
gitCommit(uploaddate, repoDir)
gitPush(repoDir)

SMTP server response: 530 5.7.0 Must issue a STARTTLS command first

I am going to share my way and it worked for me after implementing following:

Open Php.ini file and fill the all the values in the respective fields by taking ref from Gmail SMTP Settings

Remove comments from the [mail function] Statements which are instructions to the smtp Server and Match their values.

Also the sendmail SMTP server is a Fake server. Its nothing beside a text terminal (Try writing anything on it. :P). It will use gmail s,tp to send Mails. So configure it correctly by matching Gmail SMTP settings:

smtp.gmail.com
Port: 587

MVC Redirect to View from jQuery with parameters

This would also work I believe:

$('#results').on('click', '.item', function () {
            var NestId = $(this).data('id');
            var url = '@Html.Raw(Url.Action("Artists", new { NestId = @NestId }))';
            window.location.href = url; 
        })

Remove x-axis label/text in chart.js

For those whom this did not work, here is how I hid the labels on the X-axis-

options: {
    maintainAspectRatio: false,
    layout: {
      padding: {
        left: 1,
        right: 2,
        top: 2,
        bottom: 0,
      },
    },
    scales: {
      xAxes: [
        {
          time: {
            unit: 'Areas',
          },
          gridLines: {
            display: false,
            drawBorder: false,
          },
          ticks: {
            maxTicksLimit: 7,
            display: false, //this removed the labels on the x-axis
          },
          'dataset.maxBarThickness': 5,
        },
      ],

How to modify JsonNode in Java?

JsonNode is immutable and is intended for parse operation. However, it can be cast into ObjectNode (and ArrayNode) that allow mutations:

((ObjectNode)jsonNode).put("value", "NO");

For an array, you can use:

((ObjectNode)jsonNode).putArray("arrayName").add(object.ge??tValue());

What is a 'workspace' in Visual Studio Code?

Do you ever have to built a new directory and open a new Visual Studio Code window for a test project or for a feature that you want to add to your main project? Ok, so you need a workspace and enough CPU high usage...

I just wanted to mention a common usage of workspaces in Visual Studio Code in addition to all other answers.

Where can I read the Console output in Visual Studio 2015

You can run your program by: Debug -> Start Without Debugging. It will keep a console opened after the program will be finished.

enter image description here

ASP.NET Temporary files cleanup

Yes, it's safe to delete these, although it may force a dynamic recompilation of any .NET applications you run on the server.

For background, see the Understanding ASP.NET dynamic compilation article on MSDN.

How to crop an image using C#?

There is a C# wrapper for that which is open source, hosted on Codeplex called Web Image Cropping

Register the control

<%@ Register Assembly="CS.Web.UI.CropImage" Namespace="CS.Web.UI" TagPrefix="cs" %>

Resizing

<asp:Image ID="Image1" runat="server" ImageUrl="images/328.jpg" />
<cs:CropImage ID="wci1" runat="server" Image="Image1" 
     X="10" Y="10" X2="50" Y2="50" />

Cropping in code behind - Call Crop method when button clicked for example;

wci1.Crop(Server.MapPath("images/sample1.jpg"));

Find a value anywhere in a database

If you have phpMyAdmin installed use its Search feature.

Select your DataBase.

Be sure you do have selected DataBase, not a table, otherwise you'll get a completely different search dialog.

  1. Click Search tab
  2. List item Choose the search term you want
  3. Choose the tables to search

Razor-based view doesn't see referenced assemblies

In my case, I was using Razor Views outside of a web application.
Copying the dlls to my bin folder in solution solved the problem.

Proper way to catch exception from JSON.parse

This promise will not resolve if the argument of JSON.parse() can not be parsed into a JSON object.

Promise.resolve(JSON.parse('{"key":"value"}')).then(json => {
    console.log(json);
}).catch(err => {
    console.log(err);
});

Passing a dictionary to a function as keyword parameters

In python, this is called "unpacking", and you can find a bit about it in the tutorial. The documentation of it sucks, I agree, especially because of how fantasically useful it is.

Calling constructors in c++ without new

The compiler may well optimize the second form into the first form, but it doesn't have to.

#include <iostream>

class A
{
    public:
        A() { std::cerr << "Empty constructor" << std::endl; }
        A(const A&) { std::cerr << "Copy constructor" << std::endl; }
        A(const char* str) { std::cerr << "char constructor: " << str << std::endl; }
        ~A() { std::cerr << "destructor" << std::endl; }
};

void direct()
{
    std::cerr << std::endl << "TEST: " << __FUNCTION__ << std::endl;
    A a(__FUNCTION__);
    static_cast<void>(a); // avoid warnings about unused variables
}

void assignment()
{
    std::cerr << std::endl << "TEST: " << __FUNCTION__ << std::endl;
    A a = A(__FUNCTION__);
    static_cast<void>(a); // avoid warnings about unused variables
}

void prove_copy_constructor_is_called()
{
    std::cerr << std::endl << "TEST: " << __FUNCTION__ << std::endl;
    A a(__FUNCTION__);
    A b = a;
    static_cast<void>(b); // avoid warnings about unused variables
}

int main()
{
    direct();
    assignment();
    prove_copy_constructor_is_called();
    return 0;
}

Output from gcc 4.4:

TEST: direct
char constructor: direct
destructor

TEST: assignment
char constructor: assignment
destructor

TEST: prove_copy_constructor_is_called
char constructor: prove_copy_constructor_is_called
Copy constructor
destructor
destructor

Merge two rows in SQL

There might be neater methods, but the following could be one approach:

SELECT    t.fk,
          (
             SELECT t1.Field1 
             FROM   `table` t1 
             WHERE  t1.fk = t.fk AND t1.Field1 IS NOT NULL
             LIMIT  1
          ) Field1,
          (
             SELECT t2.Field2
             FROM   `table` t2 
             WHERE  t2.fk = t.fk AND t2.Field2 IS NOT NULL
             LIMIT  1
          ) Field2
FROM      `table` t
WHERE     t.fk = 3
GROUP BY  t.fk;

Test Case:

CREATE TABLE `table` (fk int, Field1 varchar(10), Field2 varchar(10));

INSERT INTO `table` VALUES (3, 'ABC', NULL);
INSERT INTO `table` VALUES (3, NULL, 'DEF');
INSERT INTO `table` VALUES (4, 'GHI', NULL);
INSERT INTO `table` VALUES (4, NULL, 'JKL');
INSERT INTO `table` VALUES (5, NULL, 'MNO');

Result:

+------+--------+--------+
| fk   | Field1 | Field2 |
+------+--------+--------+
|    3 | ABC    | DEF    |
+------+--------+--------+
1 row in set (0.01 sec)

Running the same query without the WHERE t.fk = 3 clause, it would return the following result-set:

+------+--------+--------+
| fk   | Field1 | Field2 |
+------+--------+--------+
|    3 | ABC    | DEF    |
|    4 | GHI    | JKL    |
|    5 | NULL   | MNO    |
+------+--------+--------+
3 rows in set (0.01 sec)

Renaming Column Names in Pandas Groupby function

For the first question I think answer would be:

<your DataFrame>.rename(columns={'count':'Total_Numbers'})

or

<your DataFrame>.columns = ['ID', 'Region', 'Total_Numbers']

As for second one I'd say the answer would be no. It's possible to use it like 'df.ID' because of python datamodel:

Attribute references are translated to lookups in this dictionary, e.g., m.x is equivalent to m.dict["x"]

How to connect wireless network adapter to VMWare workstation?

Use a Linux Live cd/usb and boot an that to be able to directly connect to your wifi hardware or use linux as the main OS with direct access to the wifi card and then use windows as a guest os, I know that this maybe not the ideal way but it will work.

PNG transparency issue in IE8

I use a CSS fix rather than JS to workaround my round cornered layer with transparent PNG inside

Try

.ie .whateverDivWrappingTheImage img {
background: #ffaabb; /* this should be the background color matching your design actually */
filter: chroma(#ffaabb); /* and this should match whatever value you put in background-color */
}

This may require more work on ie9 or later.

Sending a JSON HTTP POST request from Android

Posting parameters Using POST:-

URL url;
URLConnection urlConn;
DataOutputStream printout;
DataInputStream  input;
url = new URL (getCodeBase().toString() + "env.tcgi");
urlConn = url.openConnection();
urlConn.setDoInput (true);
urlConn.setDoOutput (true);
urlConn.setUseCaches (false);
urlConn.setRequestProperty("Content-Type","application/json");   
urlConn.setRequestProperty("Host", "android.schoolportal.gr");
urlConn.connect();  
//Create JSONObject here
JSONObject jsonParam = new JSONObject();
jsonParam.put("ID", "25");
jsonParam.put("description", "Real");
jsonParam.put("enable", "true");

The part which you missed is in the the following... i.e., as follows..

// Send POST output.
printout = new DataOutputStream(urlConn.getOutputStream ());
printout.writeBytes(URLEncoder.encode(jsonParam.toString(),"UTF-8"));
printout.flush ();
printout.close ();

The rest of the thing you can do it.

How to enable NSZombie in Xcode?

in ur XCODE (4.3) next the play button :) (run)

select : edit scheme

the scheme management window will open

click on the Arguments tab

you should see : 1- Arguments passed on launch 2- environment variables

inside the the (2- environment variables) place Name: NSZombieEnabled
Value: YES

And its done....

Package structure for a Java project?

The way I usually organise is
- src
        - main
                - java
                - groovy
                - resources
        - test
                - java
                - groovy
- lib
- build
        - test 
                - reports
                - classes
- doc

Connect Device to Mac localhost Server?

MacOS Sierra users can find their auto-generated vanity URL by going to System Preferences > Sharing and checking beneath the Computer Name text input. To access it, enter this URL, plus your port number (e.g. your-name.local:8000), on your iPhone over the same Wi-Fi connection as your computer.

How to remove a field completely from a MongoDB document?

In the beginning, I did not get why the question has a bounty (I thought that the question has a nice answer and there is nothing to add), but then I noticed that the answer which was accepted and upvoted 15 times was actually wrong!

Yes, you have to use $unset operator, but this unset is going to remove the words key which does not exist for a document for a collection. So basically it will do nothing.

So you need to tell Mongo to look in the document tags and then in the words using dot notation. So the correct query is.

db.example.update(
  {},
  { $unset: {'tags.words':1}},
  false, true
)

Just for the sake of completion, I will refer to another way of doing it, which is much worse, but this way you can change the field with any custom code (even based on another field from this document).

What is a "slug" in Django?

In short slug help get rid of those ugly looking urls with valid-urls for examples in an ecommerec site instead of showing the url as www.myecom.com/product/5432156 i can show it like www.myecom.com/product/iphone11 with the help of slug

check if a key exists in a bucket in s3 using boto3

Boto 2's boto.s3.key.Key object used to have an exists method that checked if the key existed on S3 by doing a HEAD request and looking at the the result, but it seems that that no longer exists. You have to do it yourself:

import boto3
import botocore

s3 = boto3.resource('s3')

try:
    s3.Object('my-bucket', 'dootdoot.jpg').load()
except botocore.exceptions.ClientError as e:
    if e.response['Error']['Code'] == "404":
        # The object does not exist.
        ...
    else:
        # Something else has gone wrong.
        raise
else:
    # The object does exist.
    ...

load() does a HEAD request for a single key, which is fast, even if the object in question is large or you have many objects in your bucket.

Of course, you might be checking if the object exists because you're planning on using it. If that is the case, you can just forget about the load() and do a get() or download_file() directly, then handle the error case there.

How to set value in @Html.TextBoxFor in Razor syntax?

I tried replacing value with Value and it worked out. It has set the value in input tag now.

@Html.TextBoxFor(model => model.Destination, new { id = "txtPlace", Value= "3" })

Setting onClickListener for the Drawable right of an EditText

Simple Solution, use methods that Android has already given, rather than reinventing wheeeeeeeeeel :-)

editComment.setOnTouchListener(new OnTouchListener() {
        @Override
        public boolean onTouch(View v, MotionEvent event) {
            final int DRAWABLE_LEFT = 0;
            final int DRAWABLE_TOP = 1;
            final int DRAWABLE_RIGHT = 2;
            final int DRAWABLE_BOTTOM = 3;

            if(event.getAction() == MotionEvent.ACTION_UP) {
                if(event.getRawX() >= (editComment.getRight() - editComment.getCompoundDrawables()[DRAWABLE_RIGHT].getBounds().width())) {
                    // your action here

                 return true;
                }
            }
            return false;
        }
    });

Push method in React Hooks (useState)?

The same way you do it with "normal" state in React class components.

example:

function App() {
  const [state, setState] = useState([]);

  return (
    <div>
      <p>You clicked {state.join(" and ")}</p>
      //destructuring
      <button onClick={() => setState([...state, "again"])}>Click me</button>
      //old way
      <button onClick={() => setState(state.concat("again"))}>Click me</button>
    </div>
  );
}

How to read and write into file using JavaScript?

To create file try

function makefile(){
  var fso;
  var thefile;

    fso = new ActiveXObject("Scripting.FileSystemObject");
    thefile=fso.CreateTextFile("C:\\tmp\\MyFile.txt",true);

    thefile.close()
    }

Create your directory in the C drive because windows has security against writing from web e.g create folder named "tmp" in C drive.

How to submit form on change of dropdown list?

Very easy to use select option submit

<select name="sortby" onchange="this.form.submit()">
       <option value="">Featured</option>
       <option value="asc" >Price: Low to High</option>
        <option value="desc">Price: High to Low</option>                                   
</select>

This code use and enjoy now:

Read More: Go Link

.NET: Simplest way to send POST with data and read response

Given other answers are a few years old, currently here are my thoughts that may be helpful:

Simplest way

private async Task<string> PostAsync(Uri uri, HttpContent dataOut)
{
    var client = new HttpClient();
    var response = await client.PostAsync(uri, dataOut);
    return await response.Content.ReadAsStringAsync();
    // For non strings you can use other Content.ReadAs...() method variations
}

A More Practical Example

Often we are dealing with known types and JSON, so you can further extend this idea with any number of implementations, such as:

public async Task<T> PostJsonAsync<T>(Uri uri, object dtoOut)
{
    var content = new StringContent(JsonConvert.SerializeObject(dtoOut));
    content.Headers.ContentType = MediaTypeHeaderValue.Parse("application/json");

    var results = await PostAsync(uri, content); // from previous block of code

    return JsonConvert.DeserializeObject<T>(results); // using Newtonsoft.Json
}

An example of how this could be called:

var dataToSendOutToApi = new MyDtoOut();
var uri = new Uri("https://example.com");
var dataFromApi = await PostJsonAsync<MyDtoIn>(uri, dataToSendOutToApi);

Why am I getting AttributeError: Object has no attribute

I have encountered the same error as well. I am sure my indentation did not have any problem. Only restarting the python sell solved the problem.